Menu Expand
Abernathy's Surgical Secrets E-Book

Abernathy's Surgical Secrets E-Book

Alden H. Harken | Ernest E. Moore

(2008)

Additional Information

Book Details

Abstract

The new edition of this leading volume in the Secrets Series® offers the very latest overview of surgical practice. A two-color page layout, question-and-answer approach, and a list of the “Top 100 Secrets” in surgery gives you the perfect concise board review or handy clinical reference, while updated coverage throughout equips you with all of the most current and essential knowledge in the field. Valuable pearls, tips, and memory aids make this the perfect resource for a fast surgical review or reference.

  • Uses bulleted lists, tables, short answers, and a highly detailed index to expedite reference.
  • Includes pearls, tips, and memory aids, making it perfect as a handy surgical review for board exams or clinical reference.
  • Covers all of today’s most common surgical procedures and techniques.
  • Presents a “Controversies” section in many chapters that highlights the pros and cons of selected procedures and approaches.
  • Features a compact trim size for enhanced portability.
  • Features revisions throughout to provide you with an up-to-date overview of today’s surgical care and practice.
  • Includes new chapters on mechanical ventilation, bariatric surgery, adrenal incidentaloma, mechanical circulatory support, and professionalism, to keep you current.

Table of Contents

Section Title Page Action Price
Front Cover Cover
Abernathy's Surgical iii
Copyright Page iv
Dedication Page v
Contents vii
Contributors xv
Preface xxiii
Top 100 Secrets 1
I. General Topics 9
Chapter 1: Are you Ready for your Surgical Rotation? 9
1. Why should you introduce yourself to each patient and ask ... 9
2. What is the correct answer to almost all questions? 9
3. Are there any simple rules from the trenches? 9
4. What is the best approach to surgical notes? 10
Hospital Discharge 13
5. What is a care transition? 13
6. What is one of the most dangerous things that you can do to your patient? 13
7. Why is a hospital discharge a dangerous procedure? 13
8. What would improve safety at discharge? 13
9. What are the most important elements of the final sign out (discharge summary)? 13
Appendix: Required Reading 14
1. Mangano DT, Goldman L: Pre-operative assessment of patients with known or suspected... 14
2. Veronesi U, Cascinelli N, Mariani L et al.: Twenty-year follow-up of a randomized study... 14
3. Fisher B, Anderson S, Bryant J et al.: Twenty-year follow-up of a randomized trial comparing total mastectomy, lumpectomy and lumpectomy... 14
4. Barnett HJ, Taylor DW, Eliasziw M et al.: Benefit of carotid endarterectomy in patients with symptomatic moderate or severe... 14
5. Endarterectomy for asymptomatic carotid artery stenosis. Executive Committee for the Asymptomatic Carotid Atherosclerosis... 14
6. Selzman CH, Miller SA, Zimmerman MA et al.: The case for beta-adrenergic blockade as prophylaxis against perioperative cardiovascular... 15
7. Van den Berghe G, Wouters P, Weekers F et al.: Intensive insulin therapy in critically ill patients... 15
8. Van De Vijver MJ, He YD, van’t Veer LJ et al.: A gene expression signature as a predictor of survival in breast cancer, N Engl J Med... 15
9. Sandham JD, Hull RD, Brant RF et al.: A randomized controlled trial of the use of pulmonary artery... 15
10. Harken AH: Enough is enough, Arch Surg 134:1061-1063, 1999 15
11. Eatock FC, Chong, P, Menezes N et al.: A randomized study of early nasogastric versus nasojejunal feeding in severe acute pancreatitis... 15
12. McFalls EO, Ward HB, Moritz TE et al.: Coronary-artery revascularization before elective major vascular surgery... 16
13. Andre T, Boni C, Mounedji-Boudiaf L et al.: Oxaliplatin, fluorouracil, and leucovorin as adjuvant treatment for colon cancer... 16
14. Fitzgibbons RJ Jr, Giobbie-Hurder A, Gibbs JO et al.: Watchful waiting vs repair of inguinal hernia in minimally symptomatic... 16
15. Neumayer L, Giobbie-Hurder A, Jonasson O et al.: Open mesh vs laparoscopic mesh repair of inguinal hernia, N Engl... 16
16. Poldermans D, Boersma E, Bax J et al.: The effect of bisoprolol on perioperative mortality and myocardial infarction in high-risk patients undergoing... 17
17. Giger UF, Michel JM, Opitz I et al.: Risk factors for perioperative complications in patients undergoing laparoscopic cholecystectomy... 17
18. Hebert P, Wells G, Blajchman M et al.: A multicenter, randomized, controlled clinical trial of transfusion requirements in critical... 17
19. The Clinical Outcomes of Surgical Therapy Study Group. A comparison of laparoscopic assisted and open colectomy for colon cancer... 18
20. Lee T, Marcantonio E, Mangione C et al.: Derivation and prospective validation of a simple index for prediction of cardiac risk... 18
21. Gurm HS, Yadav JS, Fayad P et al., for the SAPPHIRE Investigators: Long-term results of carotid stenting versus endarterectomy... 18
Chapter 2: Cardiopulmonary Resuscitation 19
1. What is cardiac arrest and sudden cardiac death? 19
2. What is the most common dysrhythmia encountered during sudden cardiac death? 19
3. What is the initial treatment for a patient found to be in ventricular fibrillation? 19
4. Is endotracheal intubation mandatory during cardiopulmonary resuscitation? 19
5. How is the airway positioned during a resuscitation attempt? 19
6. Describe the head tilt-chin lift and jaw-thrust maneuvers 19
7. What is the proper method of chest compressions in children and adults? 19
8. What is the interposed abdominal compression cardiopulmonary resuscitation technique? 20
9. What respiratory rate should be achieved during a resuscitation attempt? 20
10. What are the advantages to central line insertion during cardiac resuscitation? 20
11. Which advanced cardiac life support medications have demonstrated improved survival of neurologically intact patients at hospital discharge? 20
12. What is the sequence for treatment of ventricular fibrillation or pulseless ventricular tachycardia? 20
13. What is the sequence for treatment of asystole/pulseless electrical activity? 21
14. What are the common treatable contributing factors for cardiac arrest? 21
15. Is there a role for routine fibrinolysis in patients with pulseless electrical activity cardiac arrest? 21
16. What are the initial objectives of postresuscitation support? 21
17. What is postresuscitation therapeutic hypothermia? 21
18. What is the role of end-tidal CO2 monitoring? 22
19. What electrolyte abnormalities can lead to cardiac arrest? 22
20. What advanced cardiac life support modifications are required in patients with severe electrolyte abnormalities? 22
21. What are the common causes for cardiac arrest resulting from anaphylaxis? 22
22. What advanced cardiac life support modifications are required in patients with cardiac arrest resulting from anaphylaxis? 22
23. What advanced cardiac life support modifications are required in patients with cardiac arrest associated with trauma? 23
24. Should all patients in cardiac arrest receive cardiopulmonary resuscitation? 23
25. When should resuscitative efforts be terminated? 23
26. When should a \"slow-code\" be initiated? 23
27. Can family members be present during resuscitation of a loved one? 23
28. What are the most common causes of perioperative cardiac arrest in children? 23
29. What is the revised cardiac risk index? 23
Bibliography 24
Chapter 3: Evaluation and Treatment of Cardiac Dysrhythmias 25
1. Are cardiac dysrhythmias and cardiac arrhythmias the same? 25
2. Are all cardiac dysrhythmias clinically important? 25
3. State the goals in the treatment of cardiac dysrhythmias 25
4. How important is sinus rhythm? 25
5. Do you need to be ankle-deep in electrocardiogram paper and personally acquainted with Drs. Mobitz, Lown, and Ganong to treat cardiac dysrhythmias in the intensive care unit (ICU)? 25
6. When you are called by the ICU nurse to see a patient with an arrhythmia, what questions do you ask yourself? 25
7. If the patient requires antiarrhythmic therapy, what is the safest and most effective strategy? 26
8. In assessing a cardiac impulse, how do you distinguish supraventricular from ventricular origin? 26
9. Extra credit: Correlate the ECG with cardiomyocyte membrane ion flux 27
10. Do all wide-complex beats derive from the ventricles? 27
11. What do you do if you cannot tell whether a ventricular complex is wide or narrow? 28
12. To prevent lots of supraventricular impulses from getting to the ventricles, how do you block the atrioventricular node pharmacologically? 28
13. Why give digoxin? 28
14. Why infuse digoxin over 30 to 60 minutes intravenously? 28
15. List the steps in calling a dysrhythmia by name 28
Websites 28
Bibliography 29
Chapter 4: How to Think About Shock 30
1. Define shock 30
2. Is shock related to cardiac output? 30
3. Is organ perfusion democratic? 30
4. Is this vascular autoregulatory capacity uniform in all patients? 30
5. For diagnostic and practical therapeutic purposes, can shock be classified? 30
6. Is it advisable to treat all shock in the same sequential fashion? 30
7. What is the preferred access route for volume infusion? 31
8. Should one infuse crystalloid, colloid, or blood? 31
9. When cardiac preload is adequate, which inotropic agents are useful? 31
10. Is dopamine the same as dobutamine? 31
11. Discuss the use of dobutamine, epinephrine, and norepinephrine 31
12. When is an intraaortic balloon pump indicated? 32
13. What does an intraaortic balloon pump do? 32
14. What is diastolic augmentation? 32
15. What is systolic unloading? 33
16. Name the contraindications to intraaortic balloon pump 33
Websites 33
Bibliography 33
Chapter 5: What is Pulmonary Insufficiency? 34
1. What is pulmonary insufficiency? 34
2. How is alveolar ventilation and blood flow mismatching characterized? 34
3. How much energy is expended in the work of breathing? 34
4. Which surgical incisions most significantly compromise a patient's vital capacity? 34
5. Is a chest radiograph helpful in assessing respiratory failure? 34
6. What should you look for on the chest radiograph of a patient with impending respiratory failure? 34
7. What is adult respiratory distress syndrome? 34
8. What governs fluid flux across pulmonary capillaries into the interstitium of the lung? 35
9. What causes ARDS? 35
10. Explain high-pressure versus low-pressure ARDS 35
11. What is a normal colloid oncotic pressure (COP)? 35
12. How is COP calculated? 35
13. Define low-pressure ARDS 36
14. How can the pulmonary capillaries leak if the COP exceeds the PCWP? 36
15. What is a Lasix sandwich? 36
16. List the goals of therapy for ARDS 36
17. What governs the distribution of lung perfusion? 36
18. Discuss hypoxic pulmonary vasoconstriction (HPV) 36
19. What governs the distribution of ventilation in lung? 36
20. How does ARDS compromise lung function? 37
21. How long does it take for pulmonary arterial (deoxygenated) blood to equilibrate completely with trapped (poorly oxygenated) alveolar gas? 37
22. What is the therapy for terminal airways closure and resultant shunt secondary to the wet lung of ARDS? 37
23. When may the patient come off mechanical ventilation and be extubated safely? 37
24. What is nitric oxide? 37
25. Does inhaled NO work in ARDS? 37
Websites 38
Bibliography 38
Chapter 6: Mechanical Ventilation 39
1. Why do patients need mechanical ventilation? 39
2. Does mechanical ventilation make the lung better? 39
3. How many modes of ventilation can you name? 39
4. What three elements can characterize all of the aforementioned mechanical ventilation modes? 39
5. What are the most commonly used modes of positive-pressure ventilation? 39
6. How does assist-control ventilation work? 39
7. How does intermittent mechanical ventilation differ from assist-control ventilation? 40
8. Compare intermittent mechanical ventilation with synchronized intermittent mechanical ventilation 40
9. What are the pressure-limited types of ventilation? 40
10. Summarize the advantages and limitation of pressure-controlled ventilation 40
11. What are phase variables? 40
12. What are trigger variables? 40
13. What are limit variables? 40
14. What are the goals of mechanical ventilation in patients with acute respiratory failure? 41
15. What are the initial ventilator settings in acute respiratory failure? 41
16. Which ventilator variables control the inspiratory/expiratory (I/E) ratio? 41
17. What is positive end-expiratory pressure? 41
18. What does positive end-expiratory pressure do? 41
19. What is intrinsic or auto-positive end-expiratory pressure? 41
20. What are the side effects of positive end-expiratory pressure? 42
21. What is a ventilator bundle? 42
22. What is controlled hypoventilation with permissive hypercapnia? 42
23. What is compliance? How is it determined? 43
24. How is peak pressure measured? 43
25. How is static pressure measured? 43
26. How is compliance calculated? 43
27. Is ventilation in the prone position an option for patients who are difficult to oxygenate? 43
28. What are the indications for prone ventilation? 43
29. Junior O'Flaherty is \"fighting the ventilator.\" What do I do? 43
30. Should neuromuscular blockage be used to facilitate mechanical ventilation? 44
Bibliography 44
Chapter 7: Why Get Arterial Blood Gases? 46
1. Is breathing really overrated? 46
2. Mr. O’Flaherty has just undergone an inguinal herniorrhaphy under local anesthesia. The recovery room nurse asks permission to sedate him... 46
3. Mr. O'Flaherty is moved to the SICU, and at 2:00 am the SICU nurse calls to report that he... 46
4. You glance at the abandoned cup of coffee sitting on your well-worn copy of Surgical Secrets. What is the PO2 of that cup of coffee? 46
5. How can Mr. O'Flaherty and the coffee have the same PO2? 46
6. What is the difference between Mr. O'Flaherty's and the coffee's PO2? 46
7. What constitutes a complete set of blood gases? 46
If Mr. O’Flaherty and the coffee have the same PO2, how would Mr. O’Flaherty do if he were exchange-transfused with coffee? 46
9. Why? 47
10. How does one quantify the amount of oxygen in blood? 47
11. Why is blood thicker than coffee (or wine)? 47
12. Does the position of the oxyhemoglobin dissociation curve make any difference? 47
13. If arterial oxygen content (Cao2) or ultimately systemic oxygen delivery (cardiac output x Cao2) is what the surgeon really wants to... 47
14. What is the fastest and most practical method of increasing Mr. O'Flaherty's Cao2? 47
15. What is a transfusion trigger? 47
16. What governs respiratory drive? 48
17. How tight is respiratory control? Or, if you hold your breath for 1 minute, how much do you want to breathe? 48
18. After 60 seconds of apnea, what happens to PaCO2? 48
19. Define base excess 48
Bibliography 48
Chapter 8: Fluids, Electrolytes, Gatorade, and Sweat 49
1. What is hypertonic saline? 49
2. What is hypertonic saline good for? 49
3. Is hypertonic saline good for anything else? 49
4. How do you convert 1g of sodium into milliequivalents (mEq)? 49
5. How many mEq of sodium are in 1 teaspoon of salt? 49
6. How many mEq of sodium are in an 8-oz bottle of Gatorade? 49
7. How much does a 40-lb block of salt cost? 49
8. What is the electrolyte content of intravenous fluids? 50
9. How do these concentrations relate to body fluid and electrolyte compartments? 50
10. What are the daily volumes (mL/24 h) and electrolyte contents (mEq/L) of body secretions for a 70-kg medical student? 50
11. Are sweat glands responsive to aldosterone? Can they be trained? 51
12. Is Gatorade really just flavored athlete's sweat? 51
13. What are the daily maintenance fluid and electrolyte requirements for a 70-kg medical student? 51
14. Does the routine postoperative patient require intravenous sodium or potassium supplementation? Routine serum electrolyte testing? 51
15. Can a patient with a good heart and kidneys overcome all but the most woefully incompetent fluid and electrolyte management? 51
16. Can one throw a healthy medical student into congestive heart failure by intravenous infusion of 100ml of 5% dextrose in saline solution per kilogram per hour? 51
17. What is subtraction alkalosis? 51
18. Which electrolyte is most useful in repairing a hypokalemic metabolic alkalosis? 51
19. List the best indicators of a patient's volume status 51
20. Does a warm big toe indicate a hemodynamically stable patient? 51
21. What is the minimal adequate postoperative urine output? 51
22. What is a typical postoperative urine sodium? 51
23. Why? 52
24. Explain paradoxical aciduria 52
25. What is third spacing? 52
26. What is a Lasix sandwich? 52
Bibliography 52
Chapter 9: Nutritional Assessment, Parenteral, and Enteral Nutrition 53
Nutritional Assessment 53
1. What does a nutritional assessment include? 53
2. What are primary malnutrition and secondary malnutrition? 53
3. What is the significance of serum proteins in nutritional assessment? 53
4. How are protein requirements determined? 55
5. What is the significance of urinary nitrogen in nutritional assessment? 55
6. Should protein be severely restricted in the surgical patient with hepatic failure or renal failure? 55
7. How are kilocalorie needs determined? 55
8. What is indirect calorimetry, and when is it useful? 56
Enteral Nutrition 56
9. When should enteral nutrition be considered? 56
10. How do you access the gastrointestinal tract for feeding? 57
11. What types of enteral formulas are available? 57
12. Are specialized formulas necessary for the patient with diabetes mellitus who is critically ill? 57
13. Should specialized \"pulmonary\" formulas be used on all patients on ventilators? 57
14. What complications are related to enteral support? 57
15. Should one wait for bowel sounds or flatus before beginning enteral feedings? 57
16. Should one delay nutrition support longer in obese patients assuming they have increased reserves? 57
17. Should enteral formulas be diluted for initial presentation? 58
18. How should enteral feeding-related diarrhea be managed? 58
19. During gastric feeding, at what level of gastric residual volume (GRV) should one hold feedings? 58
20. Do enteral feedings contain enough water to meet all fluid needs? 58
21. How is enteral nutrition infused? 58
22. Is enteral nutrition better than total parenteral nutrition? 58
23. Should you discontinue enteral feeding at midnight on all patients undergoing elective surgery with general anesthesia? 59
24. Is the clear liquid diet mandatory after surgery? 59
25. Does preoperative nutrition with immune-enhancing diets improve surgical outcome? 59
26. Should actual, ideal, or adjusted body weight be used in nutrition calculations for the patient with obesity? 59
Enteral Controversies 59
27. What are probiotics, and when are they useful? 59
28. Which is more important: nitrogen or caloric balance? 59
29. Are postpyloric feedings superior to gastric feedings? 59
30. When should immune-enhancing formulas be used? 59
31. Should formula with increased fish-oil formula be used in patients who are going into acute respiratory distress syndro... 60
Parenteral Nutrition 60
32. What is parenteral nutrition? 60
33. What are the indications for parenteral nutrition? 60
34. What types of access are available for the delivery of parenteral nutrition? 60
35. Should patients with pancreatitis be exclusively fed parenterally? 60
36. Are intravenous lipids contraindicated in pancreatitis? 60
37. What complications are associated with parenteral nutrition? 61
38. Why do parenterally fed patients often develop hyperglycemia? 61
39. How should hyperglycemia be managed? 61
40. Why are intravenous fat emulsions used, and when are they contraindicated? 61
41. What is refeeding syndrome, and how is it managed or prevented? 61
42. How should parenteral nutrition be monitored? 61
43. What infusion schedules are used for TPN? 62
44. How should TPN be discontinued? 62
45. What is the cost of parenteral nutrition? 62
46. How much gut is necessary to avoid TPN dependence after small bowel resection? 62
Parenteral Controversies 62
47. Should TPN solutions contain the same percentage of fat kilocalories thatare recommended in the diet of healthy Americans (i.e., 30% of totalkilocalories)? 62
48. Does supplemental glutamine enhance outcome in surgical patients? 62
49. Should recombinant growth hormone, glutamine, and a modified diet be used routinely to maximize gut adaptation after intestinal resection? 62
Bibliography 63
Chapter 10: What Does Postoperative Fever Mean? 64
1. What is a fever? 64
2. What is malignant hyperthermia? 64
3. How is malignant hyperthermia treated? 64
4. What causes fever? 64
5. Can fever be treated? 65
6. Should fever be treated? 65
7. Should fever be investigated? 65
8. Summarize a fever work-up 65
9. What is the most common cause of fever during the early postoperative period (1 to 3 days)? 65
10. Do surgical incisions compromise spontaneous breathing patterns? 65
11. Should atelectasis be treated with incentive spirometry? 65
12. Define a wound infection 65
13. Are certain wounds prone to infection? 65
14. Do incisions become infected early after surgery? 65
15. Summarize the therapy for clostridial gas gangrene 66
16. Are nonclostridial necrotizing wound infections a cause of concern? 66
17. What are triple antibiotics? 66
18. Give the doses for triple antibiotics 66
19. Which surgical procedures are predisposed to wound infections? 66
20. When do wound infections typically occur? 66
21. How is a wound infection treated? 66
22. Is it necessary to irrigate an infected wound? 67
23. When do urinary tract infections occur? 67
24. How is a urinary tract infection diagnosed? 67
25. Name the most common late causes of postoperative fever 67
Websites 67
Bibliography 67
Chapter 11: Surgical Wound Infection 68
1. Why should we worry about surgical wound infection? 68
2. What comprises a surgical wound infection? 68
3. List the classic signs of superficial incisional, deep incisional, and organ space surgical site infections 68
4. Why do these infections occur? 68
5. Surgery always violates the skin, and we often leave foreign material. How can we avoid SSIs? 69
6. What can the surgeon do to decrease SSIs? 69
7. What else can the surgeon do to control SSIs? 69
8. Can't the surgeon predict who is going to get infected and just give them lots of antibiotics to stop infection from happening? 69
9. How do I use antibiotics correctly to prevent SSIs? 69
10. If antibiotics are used, how and when should they be administered? 70
11. Name other routes that you would use for prophylactic antibiotic administration 70
12. Does all that pulsatile lavage the surgeon uses in the operating room really do any good? 70
13. What can the patient do to help decrease SSIs? 70
14. When prevention fails, what do you do for SSIs? 71
15. What may happen with untreated superficial or deep incisional SSIs? 71
16. Define wound dehiscence 71
17. Define evisceration 71
18. What factors predispose to dehiscence? 71
19. When does wound dehiscence occur? 71
20. What are the signs and symptoms of wound dehiscence? 71
21. Describe the proper management of wound dehiscence 71
Website 72
Bibliography 72
Chapter 12: Priorities in Evaluation of the Acute Abdomen 73
1. What is the surgeon's responsibility when confronted by a patient with an acute abdomen? 73
2. Which is the most dangerous course in a patient with an acute abdomen? 73
3. Is it important to make the diagnosis in the emergency department? 73
4. If the essential goal is not to make the diagnosis, what should the surgeon do? 73
5. Are symptoms and signs uniquely misleading in any groups of patients? 73
6. Summarize the history needed 73
Physical Examination 74
7. Are vital signs important? 74
8. What is rebound? 74
9. What is mittelschmerz? 74
10. What do bowel sounds mean? 74
11. Explain the significance of abdominal distention 74
12. Is abdominal palpation important? 74
13. What is Kehr's sign? 74
14. What is a psoas sign? 74
Laboratory Studies 75
15. How is a complete blood count helpful? 75
16. Is urinalysis necessary? 75
17. What is a \"three-way of the abdomen\"? 75
18. What is a sentinel loop? 75
19. Is ultrasound valuable? 75
20. Is abdominal computed tomography valuable? 76
21. What is a double-contrast computed tomography scan? 76
Surgical Treatment 76
22. If the patient is sick (and not getting better), what should be done? 76
23. Is a negative laparotomy harmful? 76
24. Name the most challenging problem in all of medicine? 76
Website 76
Bibliography 76
Chapter 13: Surgical Infectious Disease 77
1. Have modern antibiotic developments controlled many, if not most, of the problems of surgical infection? 77
2. What kinds of barrier breech allow microbial invasion that may set up surgical site infection? 77
3. What is the difference between contamination and infection? 77
4. How can the enormous load of bacteria in the lower gastrointestinal tract be beneficial? 77
5. Whenever intraabdominal bowel spillage is encountered, is it mandatory to culture the fecal contamination and obtain sensitivities of all identified organisms? 78
6. What are preps (e.g., bowel preps)? 78
7. How is the skin or mucosal cavities of a patient sterilized to prepare a sterile field for operative incision? 78
8. What means can be used to reduce surface resident flora without further injuring the skin or mucosa? 78
9. What are \"pipe cleaner\" antibiotics? 79
10. What is selective gut decontamination? How does it work? 79
Antibiotics 79
11. Are antibiotics the classic wonder drugs? 79
12. What is meant by generations of antibiotics, as in third-generation cephalosporins? 79
13. What is the role of third-generation cephalosporins in surgical prophylaxis? 80
14. How do enzyme inhibitors combined with antibiotics enhance their antimicrobial spectrum? 80
15. What are the most expensive kinds of antibiotic therapy? 81
16. Can oral antibiotics be given in place of intravenous antibiotics in seriously ill surgical patients? 81
Prophylaxis 81
17. Should systemic antibiotic prophylaxis be used in elective colon resection? 81
18. Are two prophylactic doses better than one in preventing infection? Are three doses better still? 81
19. What factors determine the timing of antibiotic administration under the criteria of prophylaxis? 82
20. To be safe, why not administer prophylactic antibiotics to all patients undergoing any kind of operation? 82
Management Of Surgical Site Infections 82
21. What is the drug of choice for the treatment of an abscess? 82
22. Which abscess treatment is the important one in determining the outcome of a patient with intraabdominal sepsis? 83
23. Which is preferred for draining an intraabdominal abscess, a needle or a knife? 83
24. What is the role of gallium scintiscanning in early finding of abscesses in the abdomen? 83
Extra Credit Questions 83
25. Should all patients undergoing elective laparotomy receive prophylactic antibiotic coverage? 83
26. Which abscess is the most important one to be drained? 84
27. Is postoperative fever the earliest and most frequent sign of an incisional infection? 84
28. Should you begin amphotericin at the first isolation of Candida species drawn from any intravenous catheter line? 84
29. Are antibiotic drug combinations always superior to a single antibiotic agent? 84
30. Is antibody treatment of circulating endotoxin a clinically important tool? 84
31. What is the role of human recombinant activated protein C in patients with sepsis? 85
Websites 85
Bibliography 85
Chapter 14: Risks of Blood-Borne Disease 86
1. What infectious diseases are transmissible via blood transfusion? 86
2. What are the estimated risks of HBV, HCV, and HIV transmission by blood transfusion in the United States? 86
3. Which blood-borne pathogens pose a risk to surgeons? 86
4. What is the risk to healthcare workers of exposure to hepatitis B virus? 87
5. What is the risk to healthcare workers of exposure to hepatitis C virus (HCV)? 87
6. What is the risk to healthcare workers of exposure to HIV? 87
7. How well does hepatitis B vaccination protect against the disease? 87
8. Are patients at risk of infection from surgeons who are infected with HBV? 88
9. What is the proper response after percutaneous exposure to a patient with known hepatitis B? 88
10. What are the recommendations for hepatitis B immunization? 88
11. What are the recommendations for hepatitis C immunization? 88
12. Does laparoscopic surgery minimize the risk of HIV contamination? 88
13. Is double gloving an effective method of protection? 88
14. Are non-percutaneous exposures (eye splash) a major threat to surgeons? 88
15. What is the surgeons' rate of exposure to blood and body fluids? 89
16. Again, what are the seroconversion rates for HIV, HBV, and HCV exposure? 89
17. Are there effective methods to reduce the risk of transmission of blood-borne diseases to surgeons? 89
18. What is the risk to surgeons in training? 89
Bibliography 89
II. Trauma 91
Chapter 15: Initial Assessment 91
1. What is the \"golden hour\"? 91
2. Name the major components of the initial assessment of the trauma patient 91
3. What is the purpose of the primary survey? 91
4. Define the ABCDE mnemonic of the primary survey that reinforces the fact that life-threatening injuries kill in a predicable order 91
5. What are the adjuncts to the primary survey? 91
6. Identify the one concept that can prevent unexpected acute deterioration of the trauma patient during initial assessment 91
7. Name the two major causes of death during the first 24 hours after injury 91
8. How is the airway assessed? 92
9. What are the causes of upper airway obstruction in the trauma patient? 92
10. What are the initial maneuvers used to restore an open airway? 92
11. What are the indications for a definitive airway? 92
12. List the types of definitive airway that are available in their order of priority 92
13. What are the indications for a surgical airway? 92
14. How does one \"clear the C-spine\"? 92
15. What are the five non-airway conditions that pose an immediate threat to breathing in the trauma patient? 92
16. What are the preferred sites of emergent intravenous access? 93
17. What are common, simple measures of assessing hemodynamic stability in a trauma patient? 93
18. What is the Glasgow Coma Scale and what does it measure? 93
19. What fluids should be used for initial resuscitation? 93
20. What does FAST mean, and how does it help in trauma evaluation? 93
21. What is DPL, and does it have a role in trauma evaluation? 93
22. How can I learn proficiency at initial assessment? 94
Bibliography 94
Chapter 16: Posttraumatic Hemorrhagic Shock 95
1. Are hemorrhagic shock and hypovolemic shock the same? 95
2. What is hemorrhagic shock? 95
3. What is the initial management of hemorrhagic or hypovolemic shock? 95
4. Describe the cellular manifestations of hemorrhagic shock 95
5. List the clinical manifestations of hemorrhagic shock 95
6. How can blood volume be estimated in adults and children? 95
7. State the first physiologic response to hypovolemia 95
8. What are the skin manifestations? 95
9. Can the neck veins tell you anything? 96
10. Is the hematocrit a reliable guide for estimating acute blood loss? 96
11. What is the appropriate choice for intravenous solution during resuscitation? 96
12. What is base deficit, and how is it useful during resuscitation? 96
13. What are the clinical classifications of shock and the associated clinical manifestations? 96
14. What are the other types of shock, and how do they differ from hemorrhagic shock? 97
15. When should fluid resuscitation be initiated on the patient with multiple traumas? 97
16. What are the potential sources of occult blood loss when trying to ascertain apatient’s hemodynamic status? 97
17. What is the patient called who becomes unstable after the initial resuscitation,and why is it important to recognize this phenomenon? 97
18. When is blood transfusion indicated during initial resuscitation? 97
19. How does hemorrhagic shock lead to multiple organ failure? 97
Bibliography 98
Chapter 17: Traumatic Brain Injury 99
1. Is traumatic brain injury (TBI) a common problem? 99
2. What is a concussion? 99
3. How is the GCS score derived? 99
4. When should a neurosurgeon be consulted? 99
5. How do you initially assess the patient with a brain injury? 99
6. What takes priority in a patient who is hypotensive also with a traumatic brain injury? 99
7. What is the significance of anisocoria in a patient with a decreased level of consciousness? 100
8. What if the larger pupil is reactive? 100
9. Is the term \"semicomatose\" inaccurate? 100
10. How is motor response tested? 100
11. What is the significance of periorbital ecchymosis (raccoon eyes) and ecchymosis over the mastoid (Battle's sign)? 100
12. Should scalp lacerations be explored in the emergency department? 100
13. Which patients need CT scans of the head? 100
14. What are the common traumatic surgical lesions? 101
15. When is intracranial pressure monitoring indicated? 101
16. Describe the initial treatment of patients with a suspected increase in ICP 101
17. Should all patients with elevated ICP be hyperventilated? 101
18. In hemodynamically stable patients, how do you decrease ICP? 101
19. What is the end point of treatment with diuretics? 101
20. What is the significance of cerebral perfusion pressure (CPP)? 101
21. Why should all children with TBI be undressed and examined thoroughly? 102
22. Should posttraumatic seizures be treated prophylactically? 102
23. Which coagulopathy is associated with severe brain injury? 102
24. What other medical complications may result from severe head injury? 102
25. If a patient is awake with significant neurologic symptoms but no abnormality on CT scan, what are the likely explanations? 102
26. Are gunshot wounds that cross the midline of the brain uniformly fatal? 102
27. What is the significance of concussion? 102
28. Can patients with minor TBIs be discharged from the emergency department? 102
29. Is brain injury permanent? Is the outcome always poor? 102
30. What is the threshold for treating increased ICP? 103
31. Should high dose steroids be given to TBI patients to treat increased ICP? 103
32. Are patients with traumatic brain injury at risk for deep venous thrombosis and pulmonary embolus? 103
Websites 103
Bibliography 103
Chapter 18: Spinal Cord Injuries 105
1. What is the difference between a spinal injury and a spinal cord injury? 105
2. Describe the evaluation of a patient with a suspected spine injury 105
3. How do you minimize the risk of additional spine injury in hospital? 105
4. How is the level of the spinal cord injury defined? 105
5. Which type of injury is commonly associated with cervical spine injury? 105
6. How can the spinal cord be evaluated in patients with associated head injury? 105
7. Which other significant injury may mimic, at presentation, a high thoracic cord injury? 105
8. What is spinal shock? 106
9. Describe an adequate radiologic evaluation 106
10. Describe the proper way to read a lateral cervical spine film 106
11. What about the anteroposterior film? 106
12. Can a patient have a spinal cord injury and normal plain radiographs? 106
13. Is magnetic resonance imaging useful in the evaluation of acute spine trauma? 106
14. Fractures of C1 and C2 are visualized best with which view? 107
15. What is hangman's fracture? 107
16. Define deficits in complete transverse myelopathy, anterior cord syndrome,central cord syndrome, and Brown-Se'quard syndrome 107
17. What is the role of methylprednisolone in the treatment of acute cord injury? 107
18. Do patients with spinal cord injuries ever undergo acute surgery? 108
19. How is the bony injury treated? 108
20. What is the outcome in patients with spinal cord injury? 108
21. Are cervical spine injuries associated with injuries to the carotid or vertebral arteries? 108
22. Should all patients with spinal cord injuries have inferior vena cava filters placed to prevent pulmonary embolus? 108
Website 109
Bibliography 109
Chapter 19: Penetrating Neck Trauma 110
1. Why are penetrating neck wounds unique? 110
2. What constitutes a penetrating neck wound? 110
3. Which side of the neck is more likely to be injured? 110
4. Do gunshot wounds and knife wounds cause the same relative injuries? 110
5. What are the priorities in the management of penetrating neck trauma? 110
6. How should bleeding be controlled at the accident scene and in the emergency department? 111
7. Should you explore the neck wound in the trauma bay? 111
8. What physical examination findings should be elicited? 111
9. How often do patients with cervical crepitus have a significant injury? 111
10. What are the three zones of the neck? 111
11. Why are penetrating injuries divided into zones? 112
12. What are the indications for immediate operative exploration? 112
13. What is selective management of penetrating neck trauma? 112
14. Should arteriography be performed on all patients? 112
15. What is the value of other diagnostic studies, such as esophagography, esophagoscopy, laryngoscopy, and bronchoscopy? 112
16. Should an asymptomatic patient with a penetrating neck wound be sent home from the emergency department? 113
Bibliography 113
Chapter 20: Blunt Thoracic Trauma 114
1. How often do patients with isolated blunt chest trauma need an emergent operation? 114
2. In a patient with a hemothorax after blunt chest injury, what is the most important guide for the decision to operate? 114
3. What is a tension pneumothorax? 114
4. What are the clinical signs of tension pneumothorax? 114
5. How is tension pneumothorax treated? 114
6. Does it matter how many ribs are broken? 114
7. What is a flail chest? 114
8. How does flail chest impact ventilation? 114
9. Do all patients with a flail segment then need to be put on a ventilator? 115
10. Does flail chest affect oxygenation? 115
11. What is the natural history of pulmonary contusion? 115
12. What is the most common initial presentation of blunt injury to the thoracic aorta? 115
13. Of patients surviving to reach the hospital, where is the most common injury to the thoracic aorta? 115
14. What are the clinical signs of torn thoracic aorta? 115
15. What findings on chest radiograph are associated with rupture of the descending thoracic aorta? 115
16. In the stable patient with a major mechanism of injury or chest radiographs consistent with aortic injury, how is the diagnosis made? 115
17. Junior O'Flaherty was hit in the chest with a baseball bat. How can I tell if he has a bruise on his heart (myocardial contusion)? 115
18. Ok, then, how do I tell if something bad is going to happen to Junior's heart? 116
19. Where do blunt injuries to the bronchus usually occur? How do they present? 116
20. What are the indications for emergency department thoracotomy after blunt chest injury? 116
21. What is traumatic asphyxia? 116
Bibliography 116
Chapter 21: Penetrating Thoracic Trauma 118
1. How often do patients with penetrating chest wounds need an operation? 118
2. What are the indications for emergency department thoracotomy after penetrating chest wounds? 118
3. What is the \"6-hour rule\" for chest injuries? 118
4. How much blood in the pleural space can be reliably detected by chest radiograph? 118
5. If a stable patient with a penetrating chest wound has a lot of blood coming out of a chest tube, when should I operate? 118
6. What is a \"clam shell\" thoracotomy? 118
7. What is an open pneumothorax? 118
8. How is an open pneumothorax treated? 118
9. Where is \"the box,\" and why is it important? 119
10. What is Beck´s triad? Is it useful in penetrating chest injuries? 119
11. In a stable patient with suspected penetrating cardiac injury, what is the most important initial study? 119
12. Junior O´Flaherty just got stabbed in the heart. What is he likely to die of? 119
13. What is the initial therapeutic maneuver in the patient with a penetrating cardiac wound who is not yet hypotensive? 119
14. In a penetrating chest wound, how do I tell if the diaphragm is also injured? 119
15. Why is it important to detect a small diaphragmatic laceration? 119
16. Junior O'Flaherty was shot all the way through his mediastinum. He seems stable: does he need an operation? 119
17. Are prophylactic antibiotics warranted to prevent empyema after tube thoracostomy? 120
18. What is the most important risk factor for posttraumatic empyema? 120
19. What is a bronchovenous air embolism? 120
20. How is bronchovenous air embolism diagnosed and treated? 120
21. In a penetrating esophageal injury, where may air be evident on physical examination? 120
22. How do penetrating tracheobronchial injuries present? 120
23. What does a blurry bullet on a chest radiograph indicate? 120
Bibliography 121
Chapter 22: Blunt Abdominal Trauma 122
1. What elements of the history are important in evaluating a patient with suspected blunt abdominal trauma (BAT)? 122
2. Is physical examination accurate in the diagnosis of intraabdominal injury? 122
3. Which organs are most frequently injured in BAT? 122
4. What diagnostic studies are helpful in BAT? 122
5. How has the availability of ultrasound changed the initial evaluation of BAT? 123
6. How is hollow organ injury diagnosed? 123
7. What are the indications for urgent operation in a patient with BAT? 123
8. How does time in the emergency department (ED) impact the mortality of patients requiring emergent operation for BAT? 123
9. What is the role of angiographic embolization? 123
10. What is the \"bloody viscus cycle\"? 123
11. What is a staged or abbreviated laparotomy (damage control surgery)? 123
12. When is staged laparotomy used in trauma patients? 124
Website 124
Bibliography 124
Chapter 23: Penetrating Abdominal Trauma 125
1. Why is the evaluation different for patients with stab wounds versus gunshot wounds? 125
2. What are the indications for emergent laparotomy in patients with stab wounds? 125
3. What are the indications for immediate laparotomy in patients with gunshot wounds? 125
4. When is emergency department thoracotomy indicated for a penetrating abdominal wound? 126
5. What are the key elements of the secondary survey? 126
6. What are the appropriate initial studies? 126
7. What is the difference between a penetrating wound to the anterior abdomen versus the flank or back? 127
8. How is an anterior abdominal stab wound evaluated in asymptomatic patients? 127
9. What constitutes a positive diagnostic peritoneal lavage result after penetrating trauma? 127
10. How are stab wounds to the flank and back evaluated in asymptomatic patients? 127
11. How is a lower chest stab wound evaluated? 127
12. Which patients with abdominal gunshot wounds are candidates for nonoperative management? 127
13. If abdominal operative exploration is indicated, what is the general approach? 128
Controversy 128
14. What is the role of laparoscopy and thoracoscopy after penetrating abdominal trauma? 128
Bibliography 128
Chapter 24: Hepatic and Biliary Trauma 130
1. How often is the liver injured in trauma? 130
2. Do the liver and spleen respond similarly to injury? 130
3. What are the determinants of mortality after acute liver injury? 130
4. What history and physical signs suggest acute liver injury? 130
5. What diagnostic tests are helpful in confirming acute liver injury? 130
6. What is the role of hepatic angiography? 130
Surgical Anatomy of the Liver 131
7. How many anatomic lobes are present in the liver? What is their topographic boundary? 131
8. What is the blood supply to the liver and the relative contribution of each structure to hepatic oxygenation? 131
9. What are the most common variations in hepatic arterial supply to the right and left lobes of the liver? 131
10. What is the venous drainage of the liver? 132
Operative Management of Liver Injury 132
11. How are acute liver injuries classified? 132
12. Do all patients with a traumatic liver injury require surgery? 132
13. Which patients are more likely to fail nonoperative management? 132
14. What are the options for temporary control of significant hemorrhage in victims of hepatic trauma? 133
15. What is the Pringle maneuver? 133
16. What is the finger fracture technique? 133
17. What is the role of selective hepatic artery ligation in securing hemostasis in patients with a major liver injury? 133
18. Why is retrohepatic vena caval laceration lethal? 133
19. What is the physiologic rationale for use of a shunt in attempted repair of retrohepatic vena caval injuries? 133
20. What is the intrahepatic balloon tamponading device? 133
21. What are the indications for perihepatic packing? 133
22. What is the abdominal compartment syndrome? 134
23. What are the common complications related to liver injury? 134
Biliary Tract Injury 134
24. Why are complications associated with bile duct leaks? 134
25. What is the incidence of bile duct leak? 134
26. What is the initial management of a bile leak? 134
Website 135
Bibliography 135
Chapter 25: Splenic Trauma 136
1. What is the physiologic role of the spleen? 136
2. What injury patterns are associated with splenic trauma? 136
3. What are the signs and symptoms of splenic injury? 136
4. What studies can help in diagnosing splenic trauma? 136
5. How are splenic injuries classified, and why is that important? 136
6. Do splenic injuries require laparotomy? 137
7. What are contraindications to nonoperative management of splenic injuries? 137
8. What is the failure rate of nonoperative management of splenic injury? 137
9. What is delayed rupture of the spleen? 137
10. What are the general principles of operative management of the injured spleen? 137
11. What early complications arise after splenectomy? 137
12. What is splenic autotransplantation? 138
13. Does splenic autotransplantation preserve splenic function? 138
14. Does postsplenectomy leukocytosis predict infection? 138
15. Should a follow-up computed tomography scan be performed after nonoperative management of splenic injuries before patient discharge? 138
16. What is overwhelming postsplenectomy sepsis, and how is it prevented? 138
Websites 139
Bibliography 139
Chapter 26: Pancreatic and Duodenal Injury 140
1. How common are pancreatic and duodenal injuries? 140
2. What other injuries are typically associated with penetrating pancreatic trauma? 140
3. How are pancreatic injuries diagnosed preoperatively? 140
4. What are some of the commonly used surgical options for the treatment of pancreatic injuries? 140
5. Describe the common complications of pancreatic injuries 140
Duodenum 141
6. What is the role of computed tomography scanning in diagnosing blunt duodenal injuries? 141
7. What is the importance of the Kocher maneuver? 141
8. What are the four portions of the duodenum and their surgical relationships? 141
9. How are duodenal injuries classified? 141
10. What are the main surgical options for penetrating duodenal injuries? 142
Bibliography 143
Chapter 27: Trauma to the Colon And Rectum 144
Colon Trauma 144
1. How do most colon injuries occur? 144
2. How are colon injuries diagnosed? 144
3. How are colon injuries graded? 144
4. What are three primary surgical options for managing a colon injury? 144
5. What are the advantages and disadvantages of each of these options? 144
6. How are most patients with colon injuries surgically managed? 144
7. How should the surgical incision and penetrating wound be managed? 145
8. What complications are associated with colonic injury and its treatment? 145
Rectal Trauma 145
9. How do rectal injuries occur? 145
10. How are rectal injuries diagnosed? 145
11. How are patients with intraperitoneal rectal injuries treated differently from those with extraperitoneal injuries? 145
12. What are the four basic principles for managing simple extraperitoneal rectal injuries? 145
13. How are complex extraperitoneal rectal injuries managed? 145
14. What complications are associated with rectal trauma and its treatment? 145
15. What is the role of antibiotics in colorectal trauma? 146
Bibliography 146
Chapter 28: Pelvic Fractures 147
1. What are the first steps in the evaluation and treatment of a patient with pelvic trauma? 147
2. What are the sources and potential volume of bleeding in the displaced pelvic fracture? 147
3. Should a Foley catheter be placed in trauma patients with displaced pelvic fractures? 147
4. What is the incidence of urologic injury associated with pelvic fractures? 147
5. What are the commonly used radiographic classification schemes for pelvic fractures? 147
6. What is an open pelvic fracture? 147
7. When is acute mechanical stabilization of a pelvic fracture indicated? 148
8. What is pelvic packing, and when is it used? 148
9. What is the role of angiography in an acute pelvic fracture? 148
10. Why do patients die from pelvic fractures? 148
11. What is external fixation? 148
12. Is there a role for pneumatic antishock garments in the treatment of pelvic fractures? 148
13. When can patients with a pelvic fracture ambulate? 149
14. What is the most common source of arterial bleeding associated with a pelvic fracture? 149
15. Which gender and what portion of the urethra is most commonly injured in patients with a displaced pelvic fracture? 149
16. Describe the mechanism that results in a bladder rupture 149
17. What are the three radiographic views required to evaluate patients with pelvic fractures? 149
18. What is the appropriate insertion location for a diagnostic peritoneal lavage catheter in the presence of a pelvic fracture? 149
19. What percent of patients with an unstable pelvic fracture will suffer an associated neurologic injury? 149
20. What is a potential pitfall of aggressive blood transfusion of patients with hemodynamically unstable pelvic fracture? 149
21. What is the significance of an L5 transverse process fracture in a patient with a pelvis fracture? 149
Websites 150
Bibliography 150
Chapter 29: Upper Urinary Tract Injuries 151
1. What is the most common type of renal trauma in the United States, blunt or penetrating? 151
2. Do most renal injuries require surgery? 151
3. Are pediatric kidneys more susceptible to major injury? 151
4. When should potential renal trauma be investigated? 151
5. When does one suspect renal trauma? 151
6. What imaging study is the best to evaluate renal trauma? 151
7. What is a \"single-shot IVP\" and when do you perform it? 151
8. How is renal trauma classified? 152
9. How is the management according to the degree of trauma? 152
10. What are the different types of renal pedicle trauma? 152
11. How long can a nonperfused kidney tolerate warm ischemia? 152
12. What is the significance of delayed gross hematuria? 152
13. How do you manage an unexpected retroperitoneal bleeding during surgical exploration? 152
14. How are patients with posttraumatic urine extravasation managed? 152
15. What is included in conservative management of renal trauma? 152
16. What is the likelihood of subsequent hypertension? 153
17. How are most ureters damaged? 153
18. How do you evaluate and identify an ureteral injury? 153
19. What are the potential consequences of missed ureteral injury? 153
20. Distal ureter is injured and ureteral reimplantation with psoas hitch (tack up the bladder to the psoas muscle)... 153
Websites 154
Bibliography 154
Chapter 30: Lower Urinary Tract Injury and Pelvic Trauma 155
1. What are the causes of bladder injury? 155
2. What types of bladder injury may occur with blunt trauma? 155
3. What is the likelihood of a bladder injury in patients with a fracture pelvis? 155
4. How is bladder injury evaluated? 155
5. What are the retrograde cystourethrographic patterns of bladder injury? 155
6. How is bladder rupture managed? 155
7. When should urethral injury be investigated? 156
8. When a patient presents with a pelvic fracture, is concomitant urethral injury a major concern? 156
9. How is urethral injury best assessed? 156
10. How is urethral injury managed? 156
11. What are the complications of urethral injury? 156
12. What is the differential diagnosis in blunt scrotal trauma? 156
13. What is the sonographic sign of testicular rupture? 156
14. How are patients with acute testicular rupture managed? 156
15. What is the most common cause of penile fractures? 156
16. What are the physical examination findings with a penile fracture? 157
17. How are penile fractures managed? 157
18. In penile amputation injuries, how should the amputated portion of the penis be preserved for transport? 157
19. How is major scrotal skin loss managed? 157
20. A 50-year-old woman complains of urine leakage from her vagina after a hysterectomy. What is the most likely diagnosis? 157
21. What is the best time to repair a vesicovaginal fistula secondary to an uncomplicated hysterectomy? 157
Website 157
Bibliography 157
Chapter 31: Extremity Vascular Injuries 158
1. What are the \"hard signs\" of arterial injury? 158
2. What are the four ways in which an arterial injury may present? 158
3. What are the \"soft\" signs of arterial injury? 158
4. What are the symptoms of acute arterial occlusion? 158
5. What initial screening test is used to evaluate an extremity for occult vascular injury? 158
6. What are the APIs for the upper extremity and lower extremity called? 158
7. How are WBI and ABI measured, and what is considered a normal value? 158
8. What API value raises concern for arterial injury, and what is the sensitivity and specificity? 158
9. When the API value is <0.9 in an injured extremity, what should be the next diagnostic test? 159
10. What abnormalities on arteriography determine a positive test result? 159
11. What study should be performed for patients with proximity injury or soft signs (API >0.9)? 159
12. What occult vascular injuries can be detected by duplex ultrasonography? 159
13. What is a pseudoaneurysm? 159
14. What is a true aneurysm? 160
15. What is the most effective way to control arterial bleeding in an injured extremity? 160
16. What means of controlling vascular injury should be avoided? Why? 160
17. How should a patient with an extremity vascular injury be prepared and draped in the operating room? 160
18. What else should be prepared and draped for proximal extremity injuries? 160
19. What are the operative principles relative to repair of vascular injuries? 160
20. What is the best conduit to use for extremity vascular injuries if primary repair is not possible? Why? 161
21. Should injuries to major veins of the extremities be repaired? 161
22. When should injured major veins be ligated? 161
23. What complications can develop after ligation of major extremity veins? 161
24. What is a compartment syndrome? 161
25. What is the most common cause of a compartment syndrome? 161
26. What is the earliest sign of compartment syndrome after vascular repair of an extremity? 161
27. Are there any other signs of a developing compartment syndrome of an extremity? 161
28. How is the objective diagnosis of a compartment syndrome made? 162
29. What is the treatment for compartment syndrome of an extremity? 162
30. What is the result of untreated compartment syndrome? 162
31. Which are the most commonly injured arteries in the upper extremity? 162
32. Which are the most commonly injured arteries in the lower extremity? 162
33. Can a patient with an extremity arterial injury have palpable distal pulses? 162
34. What orthopedic injuries commonly have associated vascular injuries? 162
35. For an injured extremity with concomitant fracture and vascular injury, which repair should be performed first? 162
36. After reducing or fixing an extremity fracture, what must you always do? 162
37. What is the likely diagnosis in a patient with repetitive palmar trauma and finger ischemia or necrosis? 163
38. What complications can occur after angiography when a percutaneous closure device is used on the femoral artery? 163
39. What are some of the characteristics that make computed tomography angiography an effective alternative to conventional... 163
Websites 164
Bibliography 164
Chapter 32: Facial Lacerations 165
1. What distinguishes facial lacerations from other lacerations? 165
2. What factors influence treatment for the wound? 165
3. How are clean lacerations repaired? 165
4. How are dirty lacerations repaired? 165
5. What factors influence suture selection? 165
6. Which wounds are suitable for closure with tissue adhesives and steri strips? 165
7. Should eyebrows be shaved when facial lacerations are repaired? 166
8. How should crush avulsion injuries with associated skin loss be repaired? 166
9. How should bites be treated? 166
10. Should skin grafts or flaps be used for primary closure of a wound? 166
11. When are antibiotics indicated in the treatment of facial lacerations? 166
12. What determines the quality of the scar? 166
13. When should scars be revised? 166
14. What techniques are available for scar revision? 167
Controversies 167
15. What controversies exist regarding the care and repair of facial lacerations? 167
Bibliography 167
Chapter 33: Basic Care of Hand Injuries 169
1. How are hand fractures and hand injuries splinted? 169
2. What are the signs of flexor tenosynovitis? 169
3. How is flexor tenosynotivitis treated? 169
4. How and where should hand injuries be explored? 169
5. How is emergency hemostasis of injured hands achieved? 169
6. How are fingertip injuries treated? 169
7. What is the classification system for fingertip amputations? 169
8. How are nail bed injuries repaired? 170
9. What is the initial management of flexor tendon laceration? 170
10. What is the proper management of an open fracture? 170
11. What is the proper treatment for hand infection? 170
12. What is the proper management of human and animal bites? 170
13. How are injection injuries treated? 171
14. What are the most preventable causes of deformity in hand injuries? 171
15. What is the proper emergency department treatment of the patient with an amputated part? 171
16. How should the amputated part be transported to the replantation center? 171
17. What is acute carpal tunnel syndrome? 171
18. How is acute carpal tunnel treated? 171
Website 172
Bibliography 172
Chapter 34: Burns 173
1. Where do burn injuries occur? 173
2. Who is at risk of suffering burns? 173
3. What factors influence burn outcomes most profoundly? 173
4. Do any other variables influence survival? 173
5. As a single mode of injury, why do burns pose such a devastating challenge and threat to victims? 173
6. What happens locally in burn injury? 174
7. What changes occur systemically? 174
8. How can burn victims be managed in a rational way from the time of injury? 174
9. What can first responders do when witnessing a burn injury? 174
10. What actions are needed from prehospital providers (i.e., after the prehospital crew arrives, what are their priorities)? 174
11. How does the hospital-based emergency department contribute to the care of the patients with major burns? 174
12. How are burns sized? 175
13. Besides the actual skin injury, what other associated injuries may occur? 175
14. How is inhalation injury defined? 175
15. How is inhalation injury diagnosed? 176
16. What treatment has most influenced the outcome of burn victims over the past 100 years? 176
17. Why should fluid be resuscitated, and by what route? 176
18. How is fluid therapy managed? 176
19. What should be done if this treatment algorithm fails to achieve clinical improvement and patient stabilization? 176
20. How are fluid requirements calculated when there has been a delay in the initiation of therapy? 176
21. What is the best way to care for burn wounds initially? 176
22. Why and how is the depth of a burn injury graded? 177
23. When should surgical excision of the burn wound begin? 177
24. How is the excised area managed? 178
25. What is the impact of a severe burn injury on the body? 178
26. How can we best supply fuel to the metabolic furnace of the body? 178
27. What is the role of antibiotics in burn care? 178
28. How are chemical burn injuries approached? 178
29. How are patients with electrical burns managed? 178
30. After burn injuries have healed, what important issues remain to be addressed in the rehabilitation period? 178
Website 179
Bibliography 179
Chapter 35: Pediatric Trauma 180
1. What is the leading cause of death in children in the United States? 180
2. What age groups are at particular risk for traumatic death? 180
3. What primary mechanisms account for pediatric traumatic injuries? 180
4. What is the incidence of injuries by body region? 180
5. What is the overall mortality from injury in children? 180
6. What is the mortality rate of injuries by mechanism? 180
7. Are boys and girls equally susceptible to injury? 180
8. How is a child's airway different from an adult's? 181
9. What is the appropriate size of endotracheal tube to place in a child? 181
10. What if oral endotracheal intubation cannot be accomplished? 181
11. What is a child's total blood volume? 181
12. What is the first sign of significant blood loss in children? 181
13. What are signs of hypovolemic shock in children? 181
14. Is hypotension a reliable indicator of blood loss in children? 181
15. Why are children at increased risk for hypothermia during resuscitation? 181
16. What sites are preferred for venous access in children? 181
17. What if you cannot establish an intravenous line? 181
18. What are the appropriate crystalloid and blood resuscitation volumes in children? 182
19. Why are head injuries more common in children than adults? 182
20. What types of head injuries are more common in children? 182
21. Can children have significant chest trauma without rib fractures? 182
22. What types of thoracic injuries are common or uncommon in children? 182
23. What is the frequency of abdominal organ injury in blunt trauma? 182
24. How accurate is physical examination in the evaluation of pediatric blunt abdominal trauma? 182
25. What are the advantages and disadvantages of diagnostic peritoneal lavage in children? 182
26. What are the advantages and disadvantages of computed tomography in children? 183
27. Is ultrasonography effective in the evaluation of children with abdominal trauma? 183
28. Is there a reliable method to diagnose hollow visceral injury in children? 183
29. What are the \"soft signs\" of pediatric intraabdominal injury? 183
30. What should be suspected in children with seat belt or handlebar injuries? 183
31. Does the presence of hemoperitoneum in children require laparotomy? 183
32. Do all children with solid organ injuries require operative repair? 183
33. When is nonoperative management of solid organ injury in children appropriate? 183
34. What are the long-term consequences of nonoperative management of a splenic injury in children? 184
35. What are the indications for operative intervention for solid organ injuries? 184
36. What is SCIWORA? 184
37. What is the hallmark of SCIWORA? 184
38. What percentage of pediatric deaths attributed to injury are caused intentionally? 184
39. What signs are suspicious for nonaccidental trauma (NAT)? 184
40. List the characteristics of shaken baby syndrome 184
41. What fracture patterns are suspicious for NAT? 185
42. What percentage of NAT cases involves burn injuries? What are their characteristics? 185
43. What are the necessary steps in evaluation of children with suspected NAT? 185
44. How common is postinjury multiple organ failure in children? 185
45. Does the blood glucose level in pediatric trauma patients matter? 185
Bibliography 185
III. Abdominal Surgery 187
Chapter 36: Appendicitis 187
1. What is the classic presentation of acute appendicitis? 187
2. What is the pathophysiology of appendicitis? 187
3. What is the mechanism of the periumblical pain? 187
4. Where is McBurney's point? 187
5. What is McBurney's point? 187
6. Was McBurney a cop from Boston? 187
7. What are the typical laboratory findings of a patient with appendicitis? 187
8. What layers does the surgeon encounter on exposing the appendix through a Rockey-Davis incision? 187
9. Other possible signs in appendicitis include: 187
10. Who was Rockey-Davis? 188
11. What is the blood supply to the appendix and right colon? 188
12. Does surgery for appendicitis involve a risk of mortality? 188
13. What patient groups are at higher risk of death from perforated appendicitis? 188
14. What is a \"white worm\"? 188
15. What is the differential diagnosis of right lower quadrant pain? 188
16. What is an acceptable negative appendectomy rate? 188
17. What is the role of imaging in the diagnosis of acute appendicitis? 188
18. What are sonographic and CT findings suggestive of appendicitis? 188
19. Is laparoscopic appendectomy replacing the traditional approach? 189
20. What is a Meckel's diverticulum? 189
21. Can chronic diverticulitis masquerade as appendicitis? 189
22. Can a woman with a negative pregnancy test present with an ectopic pregnancy? 189
23. Can Crohn's disease initially present as appendicitis? 189
24. Is it possible to confuse appendicitis with a TOA? 189
25. Can pelvic inflammatory disease resemble appendicitis? 189
26. How does one deal with an appendiceal carcinoid tumor? 190
27. Can appendicitis be mistaken for acute cholecystitis? 190
Websites 190
Bibliography 190
Chapter 37: Gallbladder Disease 191
1. What is the prevalence of gallstones in western society for women and men 60 years of age? 191
2. What is the difference between cholelithiasis, cholecystitis, choledocholithiasis, and cholangitis? 191
3. What percentage of asymptomatic gallstones becomes symptomatic? 191
4. Should patients with asymptomatic gallstones undergo cholecystectomy? 191
5. In what groups of patients with asymptomatic gallstones is prophylactic cholecystectomy beneficial? 191
6. What is the optimal timing for laparoscopic cholecystectomy in acute cholecystitis? 191
7. What is the conversion rate from laparoscopy to the open approach in acute cholecystitis and in symptomatic cholelithiasis?... 191
8. What is the incidence of acalculus cholecystitis? 192
9. What organisms require antibiotic coverage in biliary infections? 192
10. What is the incidence of CBD injury in open and laparoscopic cholecystectomy? 192
11. How does laparoscopic intraoperative ultrasound compare with laparoscopic intraoperative cholangiography? 192
12. Does LUS or LIOC prevent CBD injuries during cholecystectomy? 192
13. What percentage of patients undergoing cholecystectomy have unsuspected choledocholithiasis? 192
14. When, if ever, should laparoscopic cholecystectomy be performed during pregnancy? 192
15. What is the prevalence of gallbladder carcinoma found incidentally during cholecystectomy? 192
16. Why is cholecystectomy increasing in the pediatric population? 192
Bibliography 193
Chapter 38: Pancreatic Cancer 194
1. What is the magnitude of the problem? 194
2. What are the histologic types of pancreatic cancer? 194
3. What are the presenting signs of pancreatic cancer? 194
4. What is the estimated survival for pancreatic cancer patients? 194
5. Why is there such a high rate of advanced disease at diagnosis? 194
6. A previously healthy 73-year-old patient presents with pruritus, dark urine, and icteric sclerae after recent overseas travel... 194
7. What is the first step in evaluating the patient? 195
8. What if an ultrasound shows dilated extrahepatic bile ducts? 195
9. What, if any, other imaging is indicated? 195
10. What is the significance of a \"double-duct\" sign? 195
11. What in the world is CA19-9? 195
12. In this case, ultrasound, ERCP, and CT scan show dilated extrahepatic bileducts, a mass in the head of the pancreas, and no obvious cause other thancancer... 195
13. We are in the operating room, the abdomen is open, and the discussion revolves around taking out the tumor. What is a Whipple procedure? 195
14. What is distal pancreatectomy? A total pancreatectomy? 196
15. Why remove gallbladder, duodenum, and stomach if the problem is in the pancreas? 196
16. How does one determine whether to perform a Whipple procedure, distal pancreatectomy, or total pancreatectomy? What is the cure rate? 196
17. What should be done if there are nodal metastases at the celiac axis or root of mesentery? 196
18. Do any other signs of inoperability exist? 196
19. A patient is found to have unsuspected spread to the celiac axis. You carry out a biliary and gastric bypass. Is there anything else you can offer the patient, either surgically or nonsurgically? 196
20. Are there any other treatments (chemotherapy, radiation therapy, pet therapy)that improve outcomes in pancreatic cancer? 197
21. With high morbidity and low cure rates, why are surgeons so eager to do Whipple procedures? 197
Websites 197
Bibliography 197
Chapter 39: Acute Pancreatitis 199
1. What are the common causes and incidence of acute pancreatitis? 199
2. What are the uncommon causes? 199
3. What are the characteristic symptoms? 199
4. What may be found on physical examination? 199
5. What is the appropriate therapy for mild to moderate pancreatitis? 199
6. Which is the better laboratory test, amylase or lipase? 199
7. What other disease states cause hyperamylasemia? 199
8. What is the significance of hypoxemia early in the course of pancreatitis? 199
9. What is the Ranson score? 200
10. How do Ranson's indices relate to mortality? 200
11. What is the limitation of using the Ranson score for predicting severity of pancreatitis? 200
12. What is necrotizing pancreatitis? 200
13. Why is it important to differentiate acute pancreatitis from necrotizing pancreatitis? 200
14. What is the optimal method for diagnosing pancreatic necrosis with or without associated infection? 200
15. When is surgery indicated in patients with acute pancreatitis? 200
16. When should antibiotic therapy be added? 201
17. What is the most common complication of acute pancreatitis? 201
18. What is the natural history of cholelithiasis following gallstone pancreatitis? 201
19. What is the natural history of alcoholic pancreatitis? 201
Bibliography 202
Chapter 40: Diagnosis and Therapy of Chronic Pancreatitis 203
1. What is chronic pancreatitis? 203
2. What is the most common cause? 203
3. Is chronic pancreatitis the result of acute pancreatitis? 203
4. What are the signs of pancreatic insufficiency? 203
5. How much of the pancreas must be destroyed before diabetes develops? 203
6. What is steatorrhea? How does one confirm the diagnosis? 203
7. Is serum amylase elevated in patients with chronic pancreatitis? 203
8. What are the complications of chronic pancreatitis? 203
9. What is a possible source of upper gastrointestinal bleeding in a patient with chronic pancreatitis? 204
10. What is the \"chain of lakes\"? 204
11. What are the treatment options for chronic pancreatitis? 204
12. What are the indications for surgery? 204
13. Which operative procedures are commonly performed? 204
14. What is the result of operative intervention? 204
Bibliography 205
Chapter 41: Portal Hypertension and Esophageal Varices 206
1. Describe the blood supply to the liver 206
2. How is portal hypertension defined? 206
3. What is hepatopetal flow? 206
4. What are the most common causes of portal hypertension? 206
5. What are schistosomiasis and Katayama fever? 206
6. How can the causes of portal hypertension be classified anatomically? 206
7. List the four major anatomic connections between the portal and systemicvenous systems 207
8. Define sinistral portal hypertension 207
9. What are the common complications of portal venous hypertension? 207
10. What impact can portal hypertension have on other organ systems? 207
11. Liver function is classified according to what system? 207
12. What is MELD? 208
13. How is MELD calculated? 208
14. How common are esophageal varices? 208
15. Is upper gastrointestinal bleeding in cirrhotic patients with documented varicesalways variceal? 208
16. Are gastric varices a common bleeding source in patients with portalhypertension? 208
17. What factors are predictive of variceal bleeding? 208
18. Does the degree of portal hypertension predict bleeding? 208
19. An initial variceal bleed is associated with what mortality and rebleeding risk? 208
20. Should selective or nonselective b-blockers be used in the treatment ofesophageal varices? 208
21. What are the major components of acute variceal bleed management? 209
22. What pharmacologic treatments are used in acute variceal bleeding? 209
23. What endoscopic treatments are used in acute variceal bleeding? 209
24. Why should antibiotics be given to cirrhotic patients admitted for GI bleeding? 209
25. What is a Sengstaken-Blakemore tube? 210
26. What are the options for preventing recurrent variceal bleeds? 210
27. How should a patient with recurrent variceal bleeds be treated? 211
28. What is transjugular intrahepatic portosystemic shunting? 211
29. Describe the basic options for surgical shunting 211
30. How can you estimate operative mortality for elective portosystemic shunting? 211
31. Is there a definitive treatment for recurrent variceal bleeding? 211
Controversy 212
32. How should a patient with known esophageal varices be treated to prevent aninitial variceal bleed? 212
Bibliography 212
Chapter 42: Gastroesophageal Reflux Disease 213
1. What symptoms suggest gastroesophageal reflux disease (GERD)? 213
2. What is the difference between heartburn and GERD? 213
3. What causes GERD? 213
4. Is hiatal hernia an essential defect in patients with GERD? 213
5. What studies are useful to diagnose GERD? 213
6. What is the initial management of a patient suspected of having GERD? 213
7. If initial treatment fails, what should be recommended? 213
8. What is the role of proton pump inhibitor (PPI) in GERD? 214
9. When should operation for GERD be recommended? 214
10. What is the goal of surgical treatment? 214
11. What procedures can accomplish this goal and how do they do it? 214
12. What are the success rates for such procedures? 216
13. What are the long-term complications of such procedures? 216
14. How can stricture from GERD be managed? 216
Controversies 216
15. Is GERD better treated in the long term by PPI therapy or Nissen fundoplication? 216
16. Should a Nissen fundoplication be performed by laparoscopy or laparotomy? 217
17. Can this disease be treated by other minimally invasive means? 217
Website 217
Bibliography 217
Chapter 43: Esophageal Cancer 218
1. What are the risk factors for developing esophageal cancer? 218
2. What is the epidemiology of carcinoma of the esophagus? 218
3. What is Barrett´s esophagus, and how does it relate to esophageal cancer? 218
4. What are the most common presenting symptoms of esophageal cancer? 218
5. What is the diagnostic work-up for patients presenting with these symptoms? 218
6. What is the anatomic distribution of esophageal cancer? 218
7. What is neoadjuvant chemotherapy? What are its advantages and disadvantages? 219
8. What are the surgical options for treatment of carcinoma of the esophagus? 219
9. What are the risks of surgery? 219
10. What is the natural history of esophageal cancer? 219
11. Describe the stages of esophageal cancer and the respective 5-year survival rate after esophagectomy 219
12. What is an \"R0\" (or \"R zero\") resection, and how does it impact survival? 219
Websites 220
Bibliography 220
Chapter 44: Acid-Peptic Ulcer Disease 221
Duodenal Ulcer Disease 221
1. What is the risk of duodenal ulcer disease? 221
2. What is the role of Helicobacter pylori in duodenal ulcer? 221
3. Is acid hypersecretion necessary for peptic ulcer disease? 221
4. What are the clinically important complications of H. pylori infection? 221
5. What is the most commonly used test for H. pylori? 221
6. What other risk factors are associated with duodenal ulcer disease? 222
7. Which endocrine disorder is associated with severe ulcer disease? 222
8. What other endocrine disorders should be screened? 222
9. What are the clinical presentations of peptic ulcer disease? 222
10. How does the location of the ulcer affect its clinical presentation? 222
11. What are the differential diagnoses of epigastric pain? 222
12. What initial test should be performed when evaluating epigastric pain of presumed gastrointestinal origin? 222
13. How are patients with duodenal ulcer treated? 222
14. What are the recurrence rates after medical therapy? 223
15. What complications are associated with medical therapy? 223
16. How should recurrent or multiple ulcers be evaluated? 223
17. How do you evaluate a borderline serum gastrin value (200 to 500 pg/ml)? 223
18. What are the indications for operative treatment of duodenal ulcers? 223
19. What operations are used to treat duodenal ulcers? 223
20. What are Billroth I and Billroth II anastomoses? 223
21. Which procedure is preferred, Billroth I or Billroth II? 224
22. What is afferent loop syndrome? 224
23. How is afferent loop syndrome prevented? 224
24. Who was Billroth? 224
25. How does alkaline or bile reflux gastritis occur? 224
26. What is selective vagotomy? 224
27. What is dumping syndrome? 224
28. What must accompany truncal vagotomy? 224
29. What is a Heinecke-Mikulicz pyloroplasty? 225
30. What is a Finney pyloroplasty? 225
31. What is a Jaboulay pyloroplasty? 225
32. What are the rates of ulcer recurrence after surgical treatment? 225
33. What is the mortality rate of these operations? 225
34. How are patients with perforated duodenal ulcers treated? 225
35. What (ulcer-specific question) should you always ask before you proceed to the operating room? 225
36. What is the long-term result after Graham closure of a perforated ulcer? 225
37. What are the complications of surgery for duodenal ulcers? 225
38. Where do ulcers recur after operation? 225
39. Why do they recur? 226
40. How do you treat pyloric stenosis? 226
Gastric Ulcer Disease 226
41. What is the most important factor in managing gastric ulcers? 226
42. How is gastric ulcer evaluated? 226
43. In patients with bleeding peptic ulcers what are the endoscopic findings suggestive of rebleeding? 226
44. How are gastric ulcers classified? 226
45. Which is the most common type of gastric ulcer? 226
46. How do benign gastric ulcers differ from duodenal ulcers? 226
47. How is H. pylori related to gastric ulcer disease? 226
48. What is a \"trial of healing\"? 227
49. What is the aim of H. pylori eradication in the setting of gastric ulcer? 227
50. How are patients with H. pylori infection treated? 227
51. Does gastric ulcer healing guarantee a benign ulcer? 227
52. What are the indications for operative therapy of benign gastric ulcers? 227
53. What is the definitive procedure used for benign gastric ulcers? 227
54. What are the options under emergent (i.e., hemorrhage or perforation) conditions? 227
55. What is the rebleeding rate if the ulcer is left in situ? 227
56. What is giant gastric ulcer? 227
57. What is Cushing's ulcer? 228
58. What is Curling's ulcer? 228
59. What is Dieulafoy's ulcer? 228
60. What is a marginal ulcer? 228
61. When does stress gastritis occur? Why? 228
62. How are patients with bleeding stress gastritis treated? 228
Website 228
Bibliography 228
Chapter 45: Small Bowel Obstruction 230
1. Name three mechanisms of bowel obstruction, and give examples and incidence of each type 230
2. What are the most common symptoms of small bowel obstruction (SBO)? 230
3. What are the pertinent questions in the patient's history? 230
4. What are the findings on physical examination? 230
5. Is a rectal examination necessary? 230
6. Where should the examiner look for obstructing hernias? 231
7. What is the most inexpensive way to confirm the diagnosis? 231
8. What other imaging studies can be used? 231
9. Which laboratory studies are indicated? 231
10. What are the initial steps in treatment? 231
11. How can I distinguish between a complete and partial obstruction? 231
12. What conditions should be included in the differential diagnosis? 232
13. What are the three types of SBO, based on bowel viability? 232
14. What are the \"five classic signs\" of strangulation? How accurate are they? 232
15. What is the mortality rate of SBO? 232
16. What operative interventions may be needed for treatment of SBO? 232
17. Describe criteria for distinguishing viable from dead bowel at the time of operation 233
18. What is the risk of development of SBO after initial laparotomy? After previous laparotomy for SBO? Which operations are associated with high rates of SBO? 233
19. What can surgeons do to decrease the risk of SBO? 233
20. What is the role of laparoscopy in SBO? 233
21. What should I consider if the patient has had Roux-en-Y gastric bypass (RYGB)? 233
22. What can be done for patients with multiply recurrent bowel obstructions for adhesions? 233
23. Name five complications associated with surgery for SBO 233
24. Name products purported to decrease adhesion formation 233
Bibliography 234
Chapter 46: Intestinal Ischemia 235
1. What is the arterial supply to the gut? 235
2. Name the potential collateral pathways between the celiac axis and SMA? SMA and IMA? Iliac and IMA? 235
3. For extra credit, for whom is the marginal artery of Drummond named? What about the arc of Riolan? 235
4. Name the common causes of acute intestinal ischemia 235
5. What is the mortality rate of patients with acute mesenteric ischemia? 235
6. What is a \"paradoxical embolus\"? 235
7. What is the diagnostic triad of acute embolic intestinal ischemia? 236
8. How does the presentation of patients with acute thrombotic occlusion differ? 236
9. Which laboratory value is diagnostic of acute intestinal ischemia? Is acidosis? 236
10. When acute intestinal ischemia is suspected, what study is diagnostic? 236
11. How do the operative findings differ in patients with atherosclerotic occlusion and patients with SMA embolism?... 236
12. What is the appropriate management of an SMA embolus? Is there a role for thrombolysis? 236
13. How is visceral ischemia of thrombotic origin managed? 237
14. Which intraoperative tests help surgeons determine bowel viability? 237
15. When the extent of bowel viability is in question, what should be done? 237
16. How much small intestine is required to maintain adequate nutrition? 237
17. Should a second-look operation be canceled because a patient improves? 237
18. What is NOMI? 237
19. How is NOMI diagnosed and managed? 237
20. If mesenteric vein thrombosis (MVT) is suspected, which test is best? 237
21. What are the risk factors for MVT? How is it treated? 237
22. What is the primary cause of chronic mesenteric ischemia? 238
23. What is the one unique risk factor for chronic mesenteric ischemia that differs from other atherosclerotic phenomena? 238
24. What are the clinical features of patients with chronic mesenteric ischemia? 238
25. How should patients with chronic mesenteric ischemia be evaluated? 238
26. What are the goals of arterial bypass in chronic mesenteric ischemia? 238
27. If mesenteric revascularization is entertained, what five essential decisions must be considered? 238
28. What is ischemic colitis? 238
29. How is ischemic colitis diagnosed and treated? What are its prognostic implications? 239
Controversies 239
30. What is celiac compression syndrome (Dunbar's syndrome)? 239
31. Which is the preferred treatment for chronic mesenteric ischemia, antegrade or retrograde visceral artery bypass? Is it necess... 239
32. What is the role of percutaneous transluminal angioplasty in chronic mesenteric ischemia? 239
Website 239
Bibliography 240
Chapter 47: Diverticular Disease Of The Colon 241
1. What is a colonic diverticulum? 241
2. What is the difference between diverticulosis and diverticulitis? 241
3. How does a diverticulum cause pain? 241
4. Where in the colon are diverticula usually located? 241
5. At what age is diverticulitis most common? 241
6. What strategy may decrease diverticulitis in patients with diverticula? 241
7. What is the best imaging test for diagnosing acute diverticulitis? 241
8. What complications can result from perforation of a colonic diverticulum? 241
9. Can diverticular disease cause bleeding? 242
10. How can the site of diverticular bleeding be localized? 242
11. When should an operation be performed for a bleeding colonic diverticulum? 242
12. If bleeding is life threatening but cannot be localized within the colon, what treatment is required? 242
13. Which three procedures may be used when perforation of the diverticulum results in an abscess? Which has the lowest operative mortality rate? 242
14. What is the clinical evidence of a vesicocolic or ureterocolic fistula after diverticular perforation? 242
15. What procedure is required to repair a vesicocolic fistula? 243
Bibliography 243
Chapter 48: Acute Large Bowel Obstruction 244
1. What are the mechanical causes of large bowel obstruction? 244
2. How is the diagnosis made? 244
3. How is the diagnosis confirmed? 244
4. What is the role of computed tomography (CT) scanning in the diagnosis of large bowel obstruction? 244
5. Why is tenderness in the right lower quadrant (RLQ) important? 244
6. Where is the obstructing cancer usually located? 245
7. What is a volvulus? Where is it located? 245
8. When is surgery indicated? 245
9. Which operation should be performed for a large bowel obstruction? 245
10. What is the role of endoluminal stenting for acute large bowel obstruction? 245
11. What are the nonmechanical causes of large bowel obstruction? 246
12. What is Ogilvie's syndrome? 246
13. What is toxic megacolon? 246
Website 246
Bibliography 246
Chapter 49: Inflammatory Bowel Disease 248
1. What two clinical entities encompass the diagnosis of inflammatory bowel disease? 248
2. Although the two diseases often overlap, they usually can be distinguished by clinical criteria. What are the major clinical differences? 248
3. What are the major radiologic differences between the two diseases? 248
4. What are the major histological differences? 248
5. Although Crohn's disease may affect the gastrointestinal (GI) tract from the pharynx to the anus, what are the most common clinical patterns of GI involvement? 248
6. Crohn’s colitis and ulcerative colitis are often difficult to distinguish clinically.What are the major differences seen at colonoscopy? 248
7. What are the major indications for surgery in Crohn's disease? 248
8. What are the major indications for surgery in ulcerative colitis? 249
9. What is the surgical treatment of ulcerative colitis? 249
10. What is the surgical procedure for an ileal stricture? What is the procedure for multiple strictures? 249
11. How do you evaluate the placement of a stoma (ostomy)? 249
12. How does one monitor a patient with UC for dysplasia? 249
13. Does IBD have a genetic basis? 249
14. What are some of the medical therapies for inflammatory bowel disease? 249
15. What is a Brooke ileostomy? 250
16. What is pouchitis, and which patients are likely to get it? 250
Controversies 250
17. Should all patients with entero-enteral fistulas secondary to Crohn’s diseasehave surgery when the fistula is discovered? 250
18. Should all patients with ulcerative colitis that is documented for 10 years,whether the disease is active or... 250
19. Is ileorectal anastomosis an acceptable operation after colectomy for ulcerative colitis? 250
20. Should we offer a total procto-colectomy and ileal pouch for patients with Crohn's colitis? 251
Bibliography 251
Chapter 50: Upper Gastrointestinal Bleeding 252
1. What is upper gastrointestinal bleeding? 252
2. What are the most common causes of upper gastrointestinal bleeding? 252
3. What is the overall mortality rate of upper gastrointestinal bleeding? 252
4. What is the most common presentation of upper gastrointestinal bleeding? 252
5. How much gastrointestinal blood loss is necessary to cause melena? 252
6. A 45-year-old man presents to the emergency department with massive hematemesis, tachycardia, and hypotension. What should the Initial approach be? 252
7. This patient stabilizes after your interventions. Is a medical history of any value in determining a cause of the bleeding? 252
8. What physical finding may be helpful in establishing the source of bleeding? 253
9. What percentage of patients with known esophageal varices are bleeding from the varices on presentation? 253
10. Does bilious or clear NGT aspirate rule out an upper GI source of hemorrhage? 253
11. What studies can be used to determine the source of bleeding? 253
12. What is the sensitivity of EGD? 253
13. How can EGD be used to control nonvariceal bleeding? 253
14. What amount of bleeding is required to see a \"Blush\" on arteriography? 253
15. What treatment options are available to control variceal bleeding? 253
16. What are the indications for surgery in patients with upper gastrointestinal hemorrhage? 253
17. What is the surgical approach to an unstable patient with a nonlocalized upper gastrointestinal bleed who does not respond to initial resuscitation? 253
18. A patient presents with hematemesis and has a remote history of an abdominal aortic aneurysm repair... 254
19. What is a Dieulafoy's ulcer? 254
20. A patient recently admitted with a traumatic liver laceration is treated nonoperatively and later develops painless hematemesis... 254
21. What are other rare causes of upper gastrointestinal bleeding? 254
Bibliography 254
Chapter 51: Lower Gastrointestinal Bleeding 255
1. Describe the initial treatment of a patient who presents with massive lower gastrointestinal (GI) bleeding 255
2. What is the next step in evaluating the patient? 255
3. What are the two most common causes of massive lower GI bleeding? 255
4. What are other potential causes of blood from the rectum? 255
5. After a thorough history and physical examination, what is the first step toward identifying the specific site of bleeding? 255
6. Name four options for localizing lower GI bleeding 255
7. Discuss the differences between sulfur colloid scan and tagged red blood cell scan 255
8. What is the role of angiography? 256
9. What therapeutic options are available with angiography? 256
10. What role should colonoscopy play in the evaluation of patient with lower gastrointestinal bleeding? 256
11. Which patients should have angiographic embolization of the bleeding site? 256
12. What is the role of vasopressin infusion? 257
13. Do lower GI hemorrhages ever spontaneously resolve? 257
14. What are the indications for operative intervention? 257
15. What is the role of blind subtotal colectomy in the management of patients with massive lower GI bleeding? 257
16. What is the most common cause of lower GI hemorrhage in the pediatric population? 257
Bibliography 257
Chapter 52: Colorectal Polyps 258
1. What are polyps? 258
2. What are the major types of polyps? 258
3. At what age do polyps develop? 258
4. Which polyps have no malignant potential? 258
5. Which polyps have malignant potential? 258
6. Are some types of polyps more frequently associated with adenocarcinoma? 259
7. What is the relationship between polyp size and risk of adenocarcinoma? 259
8. What are juvenile polyps? 259
9. How are colorectal polyps diagnosed? 259
10. What are the risks of colonoscopy? 259
11. How can one determine whether endoscopic polypectomy is adequate treatment? 259
12. What are the screening recommendations to detect polyps? 259
13. What are the screening recommendations for patients with known polyps? 260
14. Which clinical syndromes are associated with colorectal polyps? 260
15. What is the natural history of adenomatous polyposis coli? 260
16. What are the surgical treatment options for adenomatous polyposis coli? 260
17. What role do genetic defects play in the progression of colorectal polyps to adenocarcinoma? 260
18. What role do oncogenes play in the development of adenocarcinoma from adenomatous polyps? 260
Bibliography 261
Chapter 53: Colorectal Carcinoma 262
1. What are the top three causes of cancer deaths in the United States? 262
2. List a few of the presenting symptoms of patients with colorectal cancer 262
3. What options are available to evaluate a patient who has guaiac-positivestools? 262
4. List at least five risk factors for colorectal cancer 262
5. What are the current screening recommendations of the American CancerSociety for colorectal cancers? 262
6. In what part of the colon or rectum are most cancers found? 262
7. Surgical options for colorectal cancer are dependent on the tumor location.What operation should be performed for a patient with a lesion at 25 cm from theanal verge? 262
8. What about a lesion at 9 cm from the anal verge? 262
9. What about a lesion at 4 cm from the anal verge? 262
10. What is the significance of finding adenomatous polyps in a patient’s colon? 263
11. How does the surgeon prepare the patient’s colon for an operation? 263
12. What is Dukes’ staging system? 263
13. Which patients with colorectal cancer require adjuvant (postoperative)therapy? 263
Website 264
Bibliography 264
Chapter 54: Anorectal Disease 265
General Questions 265
1. What aspect of the initial patient encounter is most important in the diagnosis of anorectal disease? 265
2. What is the most common cause of painless, bright red blood per rectum? 265
3. What are the proximal and distal anatomic landmarks of the anal canal? What is its average length? 265
4. What is the anatomic and surgical significance of the dentate line? 265
5. What is the most common cause of anorectal abscess? 265
6. What are the four potential anorectal spaces used to classify anorectal abscesses? 265
7. Define fistula in ano 265
8. What is the incidence of fistula in ano after appropriate surgical incision and drainage of acute anorectal abscesses? 266
9. What is the most important factor leading to the successful surgical eradication of anorectal abscesses and fistulae? 266
10. What is Goodsall's rule? 266
11. What is a seton? 266
Anal Fissure 266
12. What is the most common location for idiopathic anal fissure? 266
13. What are the most common symptoms of anal fissure? 266
14. What is the underlying pathophysiology of fissure in ano? 266
15. What is the differential diagnosis for anal fissure, especially if atypical in location? 266
16. How do you best diagnose anal fissure? 266
17. What are the nonoperative treatment options? 266
18. What is the most common operation performed to treat intractable fissure in ano? 266
Hemorrhoids 267
19. What are hemorrhoidal tissues, and what are their normal functions? 267
20. What is the most common cause of pathological hemorrhoids? 267
21. What is the most important difference between internal and external hemorrhoids? 267
22. What are the most common complaints associated with pathologic internal hemorrhoids? 267
23. What are the most common complaints associated with external hemorrhoids? 267
24. Are there any treatment options for symptomatic internal hemorrhoids based on identifiable physical characteristics? 267
Pilonidal Disease 267
25. What is the most common clinical presentation of a pilonidal sinus? 267
26. How is acute pilonidal abscess treated? 267
27. What is definitive therapy for pilonidal disease? 267
28. How is the best way to treat the wound? 267
29. Why is pilonidal disease rare after age 40? 267
Bibliography 268
Chapter 55: Inguinal Hernia 269
1. ‘‘Groin’’ hernia refers to which three hernias? 269
2. Francois Poupart, a French surgeon and anatomist (1616–1708), described aligament that bears his name... 269
3. Franz K. Hesselbach, a German surgeon and anatomist (1759–1816), describeda triangle that is... 269
4. Sir Astley Paston Cooper, an English surgeon and anatomist (1768–1841),described a ligament bearing his name... 269
5. Antonio de Gimbernat, a Spanish surgeon and anatomist (1734–1816), hadhis interesting name attached to the lacunar... 269
6. Indirect inguinal hernia (particularly in children) and hydrocele areassociated with which congenital abnormality? 269
7. What are the diagnostic criteria for hernia in an infant or child? 269
8. What can be done to reduce an incarcerated hernia in an infant or child? 270
9. How often can incarceration be successfully reduced? What should bedone next? 270
10. What is a Bassini repair? 270
11. What is the recurrence rate with indirect and direct hernias that have beenrepaired with classic Bassini repair technique? 271
12. Describe a McVay hernia repair 271
13. For what type of hernias is the McVay Cooper’s ligament repair most useful? 271
14. What is the Shouldice repair? 271
15. What is the reported recurrence rate for the Shouldice repair? 271
16. For what type of groin hernia is the Shouldice repair not appropriate? 271
17. Describe the Lichtenstein repair 271
18. What are the advantages of using the Marlex mesh? 271
19. For what groin area is the Lichtenstein repair not appropriate? 272
20. Which type of repair is acceptable for the femoral hernia? 272
21. What is the preperitoneal or Stoppa procedure? 272
22. Where are the spaces of Retzius and Bogros? Why are they increasinglyimportant? 274
23. How tight around the spermatic cord should a surgically fashioned, internalinguinal ring be? 274
24. What is the common fascial defect of larger indirect and all direct inguinalhernias? 274
25. On examination, the femoral hernia may be confused with what otheringuinal hernia? 274
26. What is the difference between an incarcerated and a strangulated hernia? 274
27. What operation is done for an uncomplicated indirect infant hernia? 274
28. What operation is done for an uncomplicated indirect hernia in young adults? 274
29. What operation is done for an uncomplicated but sizable direct hernia in elderlyadults? 275
30. What organ systems should be reviewed with particular care in thework-up of patients with hernia (especially elderly patients with recent onset ofhernia)? 275
31. What is a sliding hernia? 275
32. What organs can be found in sliding hernias? 275
33. What are common operative and postoperative complications of hernia repairs? 275
34. What are the common sites of hernia recurrence? 275
35. How long should the patient avoid heavy lifting after a hernia repair? 276
Controversies 276
36. What are some of the anatomic issues related to inguinal hernias? 276
37. Do all hernias require urgent repair? 276
38. What are some surgical issues in the repair of inguinal hernias? 276
Bibliography 276
Chapter 56: Bariatric Surgery 277
1. My patient weighs 250 pounds (114 kg). Is he or she morbidly obese? 277
2. Is morbid obesity alone really all that morbid? 277
3. What is \"metabolic syndrome\"? 277
4. My patient has a BMI of 40. Because he or she appears so well fed, is it safe to assume his or her nutritional status and wound healing are normal? 277
5. So, if patients who are morbidly obese are sick and do not heal well, why would an otherwise rational surgeon choose to do weight loss operations? 277
6. Do patients who undergo bariatric surgery actually get healthier as they get thinner? 277
7. If patients who are morbidly obese have decreased life expectancy, do patients who get bariatric surgery actually live longer? 278
8. Some bariatric operations (like jejunoileal bypass) were abandoned because of metabolic complications. Are there some... 278
9. A Roux-en-Y gastric bypass sounds complicated. What does it entail? 278
10. Why do patients lose weight after gastric bypass? 278
11. How much do patients usually lose after gastric bypass? 278
12. Who are the best candidates for bariatric surgery? 278
13. What are the most serious complications of gastric bypass? 278
14. What is the most reliable sign of gastrojejunal leak? 278
Bibliography 279
IV. Endocrine Surgery 281
Chapter 60: Surgical Hypertension 298
1. What are the surgically correctable causes of hypertension? 298
2. Which form of surgical hypertension is most common? 298
3. What are the most common causes of renovascular hypertension? 298
4. What clinical criteria support the pursuit of investigative studies for suspected renovascular hypertension? 298
5. What is the renin-angiotensin-aldosterone system? 298
6. How do ACE inhibitors work? 299
7. Should patients with renovascular hypertension be treated medically or surgically? 299
8. When should patients with renovascular hypertension be treated with PTRA? 299
9. What findings on history and physical examination should lead to a suspicion of pheochromocytoma? 299
10. How is pheochromocytoma diagnosed? 299
11. What is the best test to localize a pheochromocytoma? 299
12. Describe the immediate antihypertensive treatment in patients with pheochromocytoma 300
13. How is primary hyperaldosteronism (Conn's syndrome) diagnosed? 300
14. Why does Cushing's syndrome or Cushing's disease cause hypertension? 300
15. What findings suggest aortic coarctation? 300
16. How does aortic coarctation cause hypertension? 300
Bibliography 301
Chapter 57: Hyperparathyroidism 281
1. What is the prevalence of hyperparathyroidism (HPT)? 281
2. What are the symptoms of hyperparathyroidism? 281
3. What is the most common cause of hypercalcemia in an outpatient as opposed to an inpatient? 281
4. What is the differential diagnosis of hypercalcemia? 281
5. What is the essential laboratory evaluation for HPT? 282
6. Describe the embryology and anatomy of the parathyroid glands 282
7. What are the indications for parathyroidectomy? 282
8. Can a patient have hyperparathyroidism and a normal parathyroid hormone level? 282
9. What is normocalcemic hyperparathyroidism? 283
10. Outline the traditional surgical strategy of an initial exploration for primary HPT 283
11. What is the pathology of primary HPT? 283
12. What minimally invasive alternatives exist to the standard operative approach? 283
13. What preoperative localization studies are available? 283
14. How is a sestamibi scan performed, and how accurate is it? 283
15. What should one do if an adenoma is not found in the usual locations? 284
16. What is the outcome of surgery for primary hypocalcemia? 284
17. What are the complications of parathyroidectomy? 284
18. What are the signs and symptoms of hypocalcemia after surgery? 284
19. How should patients with hypocalcemia be treated? 285
20. Define persistent and recurrent HPT 285
21. What is the strategy for managing patients with persistent or recurrent HPT? 285
22. Define secondary and tertiary hyperparathyroidism 285
23. What are the indications for parathyroidectomy in end-stage renal disease? 285
24. What are the options for surgical treatment of secondary and tertiary HPT? 286
25. List the endocrinopathies in MEN I and II 286
26. What is the preferred operative approach for HPT in MEN patients? 286
27. Who performed the first parathyroidectomy? 286
28. Who was Captain Martell? 286
Bibliography 286
Chapter 58: Hyperthyroidism 288
1. What are the signs and symptoms of hyperthyroidism? 288
2. What are the three most common causes of hyperthyroidism? 288
3. How should hyperthyroidism be investigated? 288
4. How is the diagnosis of Grave's disease established? 288
5. What are the three treatment options? 288
6. Which drugs are useful for the treatment of hyperthyroidism? What are their mechanisms of action? 289
7. What are the indications for and outcome of drug treatment? 289
8. What is the regimen of radioiodine treatment? 289
9. What is the outcome of radioiodine treatment? 289
10. What are the indications for thyroidectomy for hyperthyroidism? 289
11. How should patients be prepared for surgery? 290
12. What is the extent of thyroidectomy? 290
13. What is the incidence of hypothyroidism after surgery? 290
14. What is the appropriate treatment for toxic nodular goiter? 290
15. What is the appropriate treatment for hyperthyroidism resulting from thyroiditis? 290
16. What is the appropriate treatment for thyroid storm? 290
17. What surgeon won the Nobel prize for his work with thyroid disease? 290
Websites 291
Bibliography 291
Chapter 59: Thyroid Nodules and Cancer 292
1. What is the prevalence of thyroid nodules and cancer? 292
2. What is the importance of the distinction between solitary and multiple thyroid nodules? 292
3. What features of the history and physical examination indicate a higher risk of cancer? 292
4. What is the proper laboratory evaluation of a patient with a thyroid nodule? 292
5. What imaging should be done in the evaluation of a thyroid nodule? 292
6. What is the differential diagnosis of thyroid nodules? 293
7. Which single test best predicts the need for surgical intervention? 293
8. Is levothyroxine treatment useful in the management thyroid nodules? 293
9. What are the types and distribution of thyroid cancer? 293
10. What are the axioms of thyroid surgery? 293
11. Define the various types of thyroid procedures. 294
12. What is the minimal extent of thyroidectomy for a solitary thyroid nodule? 294
13. What is the surgical therapy for thyroid carcinoma? 294
14. What is the incidence of metastatic disease to the lymph nodes? 294
15. Can anything be done to identify patients with nodal disease before the initial operation? 294
16. Describe the arterial supply and venous drainage of the thyroid 294
17. Describe the anatomy of the recurrent laryngeal nerves 295
18. What defect results from injury to the RLN? 295
19. Describe the anatomy of the superior laryngeal nerve and the defect that occurs with its injury 295
20. Do patients have voice changes independent of injury to the nerves? 295
21. What is the other major complication of thyroidectomy? 295
22. What are the postoperative therapies for well-differentiated thyroid carcinoma? 295
23. What are the indications for postoperative radioiodine (I131) ablation? 295
24. What is the appropriate degree of thyroid hormone suppression of TSH? 296
25. What is the appropriate method of following patients after their initial course of treatment? 296
26. What is the appropriate management of patients with metastatic disease? 296
Websites 296
Bibliography 296
Chapter 61: Adrenal Laparoscopic Adrenalectomy Incidentaloma 302
1. What are the anatomy and secretory products of the adrenal gland? 302
2. What questions need to be considered when an adrenal tumor is identified? 302
3. What is the incidence of incidental adrenal tumors? 302
4. What is the differential diagnosis of an incidental adrenal tumor? 302
5. What is the function of aldosterone? 303
6. What is primary hyperaldosteronism? 303
7. How do you screen for hyperaldosteronism? 303
8. How do you image the patient with primary hyperaldosteronism? 303
9. What is the treatment of primary hyperaldosteronism? 303
10. What is the outcome of unilateral adrenalectomy for APA? 304
11. What are the common clinical features of pheochromocytoma? 304
12. What is the \"rule of 10\" in relation to pheochromocytoma? 304
13. What neuroendocrine disorders are associated with pheochromocytomas? 304
14. What is the workup for a pheochromocytoma? 304
15. How should a patient with a pheochromocytoma be prepared for surgery? 305
16. What is Cushing's Syndrome? 305
17. How is Cushing's syndrome different from Cushing's disease? 305
18. What are the causes of Cushing's syndrome? 305
19. How can the causes of Cushing's syndrome be classified, and which is most common? 305
20. What is the diagnosis modality and treatment for Cushing's syndrome? 305
21. What is the outcome of resection of adrenal cortisol-producing adenomas? 306
22. What is the functional evaluation of an incidentaloma? 306
23. What are the rules of resection for incidentalomas? 306
24. What imaging features of incidentalomas are suggestive of malignancy? 306
25. Which tumors or disorders commonly appear as bilateral adrenal masses? 306
26. How should adrenal cortical carcinoma be treated, and what is the primary determinant of outcome? 306
27. What is Addison's disease? 306
28. What is Addisonian crisis, and how is it treated? 307
29. What are the relative potencies of steroids? 307
30. Which U.S. President had Addison's disease? 307
Bibliography 307
V. Breast Surgery 309
Chapter 62: Breast Masses 309
1. What are the three parts of breast screening that assist in the early diagnosis of breast cancer? 309
2. When should routine mammography begin? 309
3. Does a normal or negative mammogram guarantee that no cancer is present? 309
4. What is the role of screening magnetic resonance imaging as an adjunct to mammography? 309
5. What is the role of ultrasound in the diagnosis of breast cancer? 310
6. What is the difference between a screening and a diagnostic mammogram? 310
7. How are mammographic abnormalities characterized? 310
8. Which biopsy techniques aid in the diagnosis of mammographic abnormalities? 311
9. What are the characteristics of a dominant breast mass? 311
10. What are the four most frequently encountered palpable breast masses? 311
11. What are the differential characteristics of the most common palpable masses? 311
12. A 32-year-old woman presents with the complaint of a breast lump. Which questions about the patient´s history are important in the evaluation of the mass? 312
13. The mass identified in question 10 is discrete, not tender, easily palpable, and has gradually increased in size. What is the most appropriate next step? 312
14. How is a cyst aspiration performed? 312
15. What techniques are available for diagnosis of a palpable, solid breast mass? 312
16. What is the role for breast imaging in the evaluation of a palpable breast mass? 312
17. What is the \"triple-negative test\" or \"diagnostic triad\"? 313
Bibliography 313
Chapter 63: Primary Therapy for Breast Cancer 314
1. How is breast cancer diagnosed? 314
2. What are the limitations of needle sampling? 314
3. How do fine needle aspiration and core needle biopsy differ? 314
4. Why should the breast be imaged before performing a surgical breast biopsy? 314
5. Does a delay between biopsy and definitive treatment adversely affect cure? 315
6. How is breast cancer staged? 315
7. Why is staging of breast cancer important? 315
8. What is the overall survival rate after definitive multimodality treatment? 316
9. What is the difference between noninvasive (in situ) and invasive breast cancers? 316
10. Where does invasive breast cancer spread (other than to lymph nodes)? Which diagnostic tests are useful for identifying such metastases? 316
11. Which tests should be obtained before surgery to screen for metastases? 316
12. What are the alternatives for primary surgical treatment of invasive breast cancer? 316
13. What is the National Surgical Adjuvant Breast and Bowel Program? 317
14. What is the significance of the NSABP B-06 trial? 317
15. What is the difference among quadrantectomy, lumpectomy, and partial mastectomy? 317
16. Are some patients poor candidates for breast conservation therapy? 317
17. What is oncoplastic surgery? 318
18. After mastectomy, which patients may undergo immediate breast reconstruction (i.e., during the same operation)? 318
19. When is chest wall radiation therapy indicated after mastectomy? 318
20. What is sentinel lymph node mapping for breast cancer? 318
21. Are there risks of axillary staging by sentinel lymph node mapping? 318
22. Which tests should be obtained after surgery to screen for metastases or as baseline studies for future comparison? 318
23. What is \"neoadjuvant\" therapy for breast cancer? 319
24. What is \"inoperable\" breast cancer? 319
25. How is DCIS treated? 319
26. Can some cases of DCIS be treated by lumpectomy without radiotherapy? 319
27. How does DCIS management differ from that for lobular carcinoma in situ (LCIS)? 320
28. Why are patients with LCIS not treated surgically? 320
29. Can drugs be used to prevent breast cancer among women at high risk? 320
Website 321
Bibliography 321
IV. Other Cancers 323
Chapter 64: What Is Cancer? 323
1. What is a neoplasm? 323
2. What kinds of cancers are there? 323
3. What about skin cancers? 323
4. Why is cancer bad for you? 323
5. Are all cancers life threatening? 323
6. How do cancers start? 323
7. Is this process the same for all cancers? 323
8. How is cancer diagnosed? 323
9. What is immunohistochemistry? 324
10. What is a metastasis? 324
11. Do all cancers spread? 324
12. Does this process have an effect on how surgeons treat patients with cancer? 324
13. Why are lymph nodes removed during cancer operations? 324
14. What is a sentinel lymph node? 324
15. Do solid tumors spread in an orderly way? 324
16. How do these different models of cancer affect treatment? 325
17. Do we know which model is correct? 325
18. How else can solid tumors be treated with curative intent? 325
19. What is adjuvant therapy? 325
20. What is neoadjuvant therapy? 325
21. What cancer treatments are available in addition to surgery, radiation therapy, and cytotoxic chemotherapy? 325
22. What is targeted therapy? 326
23. Does the body fight cancer on its own? 326
24. What is a tumor-infiltrating lymphocyte? 326
25. What are palliative treatments? 326
26. What are some examples of palliative procedures? 326
27. What is cytoreductive surgery? 326
Controversy 326
28. Is axillary lymph node treatment for breast cancer of therapeutic value, or doesit merely help select patients who should receive chemotherapy? 326
Bibliography 327
Chapter 65: Melanoma 328
1. What is melanoma? 328
2. What is the incidence of melanoma? 328
3. What are the types of moles? Which are most prone to malignant change? 328
4. What are the risk factors in melanoma formation? 328
5. Which skin lesions often mimic a primary melanoma? 328
6. What is the familial melanoma syndrome? 329
7. What are common sites of melanoma development? 329
8. Where is the incidence of melanoma the highest in the world? 329
9. What are the warning signs of melanoma? 329
10. What are the types of melanoma and their incidence? 329
11. Which moles should be considered for removal? 329
12. How should suspicious nevi be biopsied? 329
13. Do melanomas spontaneously regress or even disappear? 329
14. What are the Breslow and Clark classifications of melanoma invasion? 329
15. What is the tumor, node, metastasis staging system for melanoma? 331
16. What are the chances of nodal and systemic spread of the various degrees of melanoma invasion? 332
17. What are the characteristics of a subungual melanoma? 332
18. Describe the technique of sentinel lymph node biopsy 332
19. How is sentinel lymph node biopsy changing the treatment of melanoma? 333
20. Does elective lymph node dissection improve cure rates in patients with melanoma? 333
21. What is the accuracy of sentinel lymph node biopsy for melanoma? 333
22. What features of melanoma are unfavorable for prognosis and metastatic risk? 333
23. If indicated, which types of node dissection should be performed? 333
24. What margin is appropriate for treating a primary melanoma? 333
25. Are there other treatments that improve survival in melanoma patients? 333
26. What about immunotherapy? 333
27. Can radiotherapy be helpful in melanoma treatment? 333
28. When should amputation be used in the management of locally advanced melanoma? 333
29. What is isolation limb perfusion? How is it used in melanoma? 334
30. What is the treatment of a patient with metastatic nodes confined to a single area when the primary site is unknown? 334
31. Now you´ve done it. That patient with a lymph node dissection has developed lymphedema. How do you manage it? 334
32. What should you do in the follow-up care of patients undergoing curative surgery for melanoma? 334
33. Is there a role for surgery in patients with stage IV (metastatic) melanoma? 334
Websites 334
Bibliography 334
Chapter 66: Parotid Tumors 336
1. Describe the location and characteristics of the parotid gland. 336
2. What is the relationship of the facial nerve to the parotid gland? 336
3. What branch of the facial nerve follows the parotid duct along its course, and what is its significance? 336
4. What is the salivary gland unit? 336
5. What is the significance of the salivary gland unit in tumor development? 336
6. What are the four most common benign tumors of the salivary gland origin and their characteristics? 337
7. What is the treatment of benign tumors of the parotid gland? 337
8. What is the role of intraoperative facial nerve monitoring in parotid gland surgery? 337
9. What is the significance of a \"dumbbell tumor\"? 337
10. Of all the three paired major salivary glands, which gland has the highest incidence of salivary gland neoplasms? 337
11. What is the work-up for a mass in the parotid space? 337
12. Why is the significance of facial nerve weakness or paralysis in association with parotid gland enlargement? 338
13. Describe the five most common malignant parotid tumors and their characteristics? 338
14. What is the current classification of malignant parotid gland tumors? 338
15. How are parotid tumors managed? 339
17. What are the potential complications of parotid gland surgery? 339
18. What is the role of fine needle aspiration biopsy in the diagnosis of parotid gland enlargement? 340
19. What parotid gland malignancy has a high incidence of perineural invasion? 340
20. Why should you be careful when dealing with cystic lesions of the parotid gland? 340
21. Are intraoperative frozen sections reliable to differentiate between benignand malignant parotid tumors, and would you resect the facial nerve on thebasis of a frozen section? 340
22. Is there a role for chemotherapy in the treatment of parotid gland malignancies? 340
23. Is there a role of immuno-histochemical staining in parotid gland tumor identification? 340
24. Is there a role of immuno-histochemistry in predicting survival in parotid gland malignancies? 341
Bibliography 341
Chapter 67: Neck Masses 342
1. A 34 year-old man presents with an ipsilateral 2- to 3-cm mass just below the angle of the mandible. What is the differential diagnosis? 342
2. Normal anatomy? 342
3. Does this patient seem awfully young for metastatic cancer? 342
4. Is there any way to narrow this list of possibilities? 342
5. Why not just remove the mass or lymph node to see what it is? 343
6. A complete head and neck examination shows nothing abnormal, but fineneedle aspiration of the mass reveals squamous cancer in a lymph node.What should be done next? 343
7. Is this a bit much? 343
8. Why carry out a head and neck examination in the office if I am going to proceed to triple endoscopy anyway? 343
9. Should I get a CT or MRI scan? 343
10. All that is done, and I still cannot find a primary tumor. What now? 343
11. What if the primary tumor never shows up? Does this influence survival? 343
12. If the mass or enlarged node is in the posterior triangle of the neck, is the work-up still the same? 343
13. What if fine needle aspiration of the nodes shows only lymphocytes or shows adenocarcinoma? 343
14. Lumps in the neck are common, and relatively few patients have cancer. Is this type of work-up overkill and too expensive? 344
Bibliography 344
Chapter 68: What is Atherosclerosis? 345
1. Do you have to be old to have atherosclerosis? 345
2. What is a fatty streak? 345
3. What is a foam cell? 345
4. Describe the progression of atherosclerosis. 345
5. Of 100 medical student volunteers, how many have significant atherosclerosis? 345
6. What are the classic risk factors for atherosclerotic cardiovascular disease? 345
7. How do such diverse risk factors produce similar disease? 346
8. What is the response to injury? 346
9. What is C-reactive protein? Is it just another random, nonclinically relevant marker of inflammation? 346
10. Does vascular injury mean only direct physical injury, as with an angioplasty catheter? 346
11. Are lipids important? 346
12. What is the metabolic syndrome? 346
13. What is leptin? What is its association with atherosclerosis? 347
14. Why would vitamin E be (even theoretically) protective against cardiovascular disease? 347
15. What is homocysteine? 347
16. How does homocysteine rank as a risk factor for atherosclerosis? 347
17. Should everyone take folate supplements? 347
18. What microorganisms have been implicated in atherosclerosis? 347
19. Are individuals with sexually transmitted diseases at greater risk for cardiovascular disease? 348
20. Is there an H. pylori peptic ulcer equivalent in atherosclerosis? Should we all take a macrolide a day? 348
21. If you have multiple cavities, should you electively schedule your coronary artery bypass surgery? 348
22. What is the role of the endothelium? 348
23. What are some of the products of endothelial cells that govern vasomotor tone? 348
24. What is the importance of vascular thrombosis? 348
25. Describe the three main phases of platelet involvement with thrombus formation. 348
26. What is the mechanism of plaque rupture? 349
27. What are some of the clinical complications of atherosclerotic plaque formation? 349
28. If atherosclerosis is an inflammatory disease, should we all be taking an aspirin a day? 349
Bibliography 349
Chapter 69: Arterial Insufficiency 350
1. Describe claudication and its physiology 350
2. List the different nonoperative therapies for intermittent claudication 350
3. Define critical limb ischemia 350
4. What is the ankle brachial index (ABI)? 350
5. Describe the natural history of claudication 350
6. Describe the natural history of critical limb ischemia 351
7. What are segmental limb pressures? How are they used? 351
8. Describe the natural history of graft occlusions 351
9. What is the prognosis of young patients with vascular disease? 351
10. Describe the anatomic distribution of vascular disease in diabetes 351
11. What are the implications of renal failure on outcomes? 351
12. Discuss the concept of inflow versus outflow 352
13. What are the choices for autogenous conduits? 352
14. What are the indications for arteriography? 352
15. What are the patency rates of inflow procedures? 352
16. What are the patency rates of infrainguinal bypass procedures? 352
17. Name the primary cause of perioperative mortality 353
18. Name the primary cause of perioperative morbidity 353
19. What are the causes of graft failure? 353
20. What therapeutic options are available for graft failure? 353
21. What method of graft surveillance should be used? 353
22. What therapeutic options are available for graft stenoses? 353
23. What is the role of iliac angioplasty and stenting? 354
24. How is viability determined in cases of acute ischemia? 354
25. How is thrombus distinguished from embolus in acute ischemia? 354
26. When is thrombolysis indicated? 354
27. What is compartment syndrome? 354
28. What is the role of endovascular therapy in infrainguinal occlusive disease? 354
Websites 355
Bibliography 355
Chapter 70: Carotid Disease 356
1. What primary diseases affect the carotid arteries? 356
2. What are the histological features of atherosclerotic plaques? 356
3. What are the clinical sequelae of atherosclerotic disease? 356
4. What are the most common symptoms of carotid artery disease? 356
5. Define TIA, CVA, and amaurosis fugax 356
6. What are Hollenhorst plaques? 356
7. What mechanisms produce neurologic deficits? 357
8. What is the natural history of a TIA? 357
9. What is the effect of medication on TIAs and stroke? 357
10. What does a carotid bruit signify? 357
11. Does the sound of a bruit correlate with the degree of stenosis? 357
12. What preliminary test should be ordered to evaluate a cervical bruit or carotidstenosis based on clinical findings such as TIA or CVA? 357
13. When is intervention indicated for symptomatic carotid artery disease? 357
14. Should a patient with asymptomatic stenosis undergo surgery? 358
15. What are the complications of carotid endarterectomy? 358
16. Which cranial nerves may be injured during CEA? What are the clinical signs of injury? 358
17. What is the danger of wound hematoma after surgery? 358
18. When do neurologic events occur during CEA? 358
19. What is a shunt? When is it used? 358
20. What is stump pressure? 359
21. Does stenosis recur after carotid endarterectomy? 359
22. What is the most common complication associated with reoperation endarterectomy? 359
23. In which layer of the artery is the carotid endarterectomy performed? 359
24. What anatomic landmark is useful in identifying the level of the carotid artery bifurcation? 359
25. How many branches of the internal carotid artery are located in the neck? 359
26. When the internal carotid artery is occluded, which branches of the externalcarotid artery form collaterals and reestablish circulation in the circle of Willis? 359
27. What are the functions of the carotid sinus and the carotid body? 359
28. When was the first successful surgical procedure of the extracranial carotidartery performed? Who is credited with it? 359
Controversy 359
29. What is the role of carotid artery stenting? 359
Bibliography 360
Chapter 71: Abdominal Aortic Aneurysm 361
1. What is an abdominal aortic aneurysm (AAA)? 361
2. What is the incidence of AAA? 361
3. What is the etiology of AAA? 361
4. Do AAAs have a genetic component? 361
5. Are patients with AAA prone to aneurysms in other vascular beds? 361
6. Can AAAs reliably be detected on physical examination? 361
7. Can AAAs be detected by radiography? 361
8. Which imaging method is the best for screening patients for AAA? 361
9. What is the best single imaging modality to plan AAA repair? 362
10. What is the manifestation of a symptomatic AAA? 362
11. What is the appropriate management of a patient suspected of a ruptured AAA? 362
12. Should all patients presenting with AAA rupture undergo repair? 362
13. Do all patients with ruptured AAAs make it to surgery? 362
14. How is a ruptured AAA treated operatively? 362
15. How should patients with symptomatic nonruptured AAAs be managed? 362
16. Are there any alternatives to open surgical repair for ruptured AAA? 362
17. What are the rupture rates of AAAs? 362
18. How fast do AAAs enlarge? 363
19. When are angiograms helpful in the diagnostic workup for AAA? 363
20. What is the difference between extraperitoneal and transabdominal approach? 363
21. What are endografts? Are they durable? 363
22. What are the advantages and disadvantages of an endovascular repair of an AAA (EVAR)? 363
23. What are the complications of EVAR? How are they treated? 363
24. Describe the evaluation needed for a patient receiving EVAR 364
25. What are the technical aspects of EVAR? 364
26. At what size should asymptomatic AAAs be repaired electively? 364
27. What are the technical aspects of AAA surgery? 364
28. What are the major noncardiac complications of AAA repair? 364
Websites 365
Bibliography 365
Chapter 72: Venous Disease 366
1. Where does deep venous thrombosis (DVT) originate? 366
2. What is the usual source of a pulmonary embolus? 366
3. What is Virchow's triad? 366
4. What are the major hypercoagulable syndromes (thrombophilia)? 366
5. What causes venous intimal injury? 366
6. What causes stasis of venous blood flow? 366
7. What are the usual clinical risk factors for DVT? 366
8. What signs and symptoms suggest DVT? How can DVT be accurately diagnosed? 366
9. Is there any value to D-dimer testing? 367
10. What methods of perioperative DVT prophylaxis should be used? In which surgical patients? 367
11. How does heparin work? 367
12. What is LMWH? 367
13. Should the placement of an inferior vena cava (IVC) filter ever be considered? 367
14. How long should anti-coagulation be continued post-DVT? 367
15. What are the characteristics of chronic venous insufficiency and postphlebitic or postthrombotic syndrome? 368
16. Do all patients with DVT develop postphlebitic or postthrombotic syndrome? 368
17. How are patients with postphlebitic syndrome treated? 368
18. Distinguish between phlegmasia alba dolens and phlegmasia cerulea dolens 368
19. What is venous claudication? 368
20. How can one distinguish primary varicose veins from secondary varicose veins? 369
21. Why do people develop primary varicose veins? 369
22. How, when, and in whom should varicose veins be treated? 369
Website 369
Bibliography 369
Chapter 73: Noninvasive Vascular Diagnostic Laboratory 371
1. What is the role of the vascular diagnostic laboratory (VDL) in the assessment and treatment of patients with suspected vascular disease? 371
2. What differentiates the VDL from diagnostic radiology and ultrasound? 371
Cerebrovascular Disease 371
3. Which noninvasive tests should be used to diagnose extracranial carotid artery disease? 371
4. What is duplex ultrasound? 372
5. Why is blood velocity important in assessing the degree of carotid artery stenosis? 372
6. What are the velocity criteria and categorical ranges of carotid artery stenosis? 372
7. Is duplex ultrasonography capable of determining whether the internal carotid is occluded? 372
8. How accurate is duplex ultrasound of the internal carotid if the contralateral internal carotid is occluded? 373
Venous Disease 373
9. What noninvasive test is used to diagnose acute DVT? 373
10. Can duplex ultrasound be used for surveillance in patients at high risk for DVT? 373
11. Which veins are anatomically deep veins, and which veins are superficial veins? 373
12. What noninvasive tests are useful for evaluation of venous incompetence? 373
Peripheral Arterial Occlusive Disease 374
13. What is the primary test for diagnosis of lower extremity ischemia? 374
14. What is gained by measuring pressures at limb levels other than the ankle? 374
15. What tests are used for assessing peripheral artery disease in patients whoare diabeticwhomay have incompressible arteries caused bymedial calcification? 374
16. How should the patient with suspected intermittent claudication be evaluated? 374
Duplex Ultrasound Surveillance of Vascular Therapy 374
17. What is the importance of duplex ultrasound surveillance of autogenous lower extremity bypass grafts? 374
18. Does duplex ultrasound have any role in surveillance of infrainguinal revascularization? 374
19. What is the role for surveillance duplex ultrasound following carotid endarterectomy (CEA)? 375
Controversies 375
20. Can carotid endarterectomy be performed on the basis of duplex study alone? 375
21. Does duplex ultrasound have any role in the preoperative evaluation of peripheral vascular disease? 375
22. What is the potential adverse affects of magnetic resonance angiography in patients with renal insufficiency? 376
23. What are the disadvantages of duplex ultrasound? 376
24. Should D-dimer blood tests be required before patients are evaluated by ultrasound for DVT? 376
25. Is there a therapeutic component to the noninvasive vascular diagnostic laboratory? 376
26. What is the role of duplex ultrasound in the treatment and management of abdominal aortic aneurysms (AAA)? 376
Bibliography 377
VIII. Cardiothoracic Surgery 379
Chapter 74: Coronary Artery Disease 379
1. What is angina, and what causes it? 379
2. How is angina treated? 379
3. What are the indications for coronary artery bypass graft? 379
4. What is done during a \"traditional\" CABG procedure? 380
5. What is an off-pump CABG (OPCAB)? 380
6. Why would one choose an OPCAB instead of a traditional CABG? 380
7. Does CABG improve myocardial function? 380
8. Is CABG helpful in patients with congestive heart failure? 380
9. Is CABG valuable in preventing ventricular arrhythmias? 380
10. What is the difference between PCI and CABG? 381
11. What is the rule of thumb for vessel patency? 381
12. What operative and technical problems are associated with CABG? 381
13. What are the risks of CABG? Which comorbid factors increase the operative risk for CABG? Why are large databases useful for th... 382
14. What steps are taken if a patient cannot be weaned from CPB? 382
Controversies 382
15. Is there an advantage to surgical revascularization with all arterial conduits? 382
16. What are the options for a patient with continued angina who is deemed not suitable for CABG? 382
17. What therapy should I offer to a 65-year-old male with diabetes mellitus, stable lifestyle limiting angina... 383
Website 383
Bibliography 383
Chapter 75: Mitral Stenosis 384
1. What causes mitral stenosis? 384
2. Which gender most commonly gets mitral stenosis? 384
3. What are the physical findings of mitral stenosis? 384
4. How is the diagnosis confirmed? 384
5. What is the Gorlin formula? 384
6. What is the normal size of the mitral valve? 384
7. How is the mitral valve area determined by echocardiogram? 384
8. What is the pathophysiology of mitral stenosis? 384
9. What is the main symptom of mitral stenosis? 384
10. What hemodynamic conditions precipitate symptoms in patients with mitral stenosis? 385
11. What complications may result from mitral stenosis? 385
12. Why does mitral stenosis cause pulmonary hypertension? 385
13. What is the medical therapy of mitral stenosis? 385
14. What is the natural history of mitral stenosis? 385
15. What are the indications for mechanical intervention in mitral stenosis? 385
16. What is the procedure of choice for mitral stenosis? 385
17. Which patients may be appropriate for balloon valvuloplasty? 385
18. What are the results of balloon valvuloplasty? 385
19. Which operations may be done for mitral stenosis? 386
Bonus Question 386
20. What is the Lutembacher syndrome? 386
Website 386
Bibliography 386
Chapter 76: Mitral Regurgitation 387
1. List the causes of mitral regurgitation 387
2. What is the pathophysiology of mitral regurgitation? 387
3. What are the symptoms of mitral regurgitation? 387
4. What determines left atrial pressure in mitral regurgitation? 387
5. Why does acute mitral regurgitation cause severe symptoms? 387
6. What hemodynamic conditions exacerbate mitral regurgitation? 387
7. What is the murmur of mitral regurgitation? 387
8. How is the diagnosis confirmed? 387
9. What is the medical therapy for mitral regurgitation? 388
10. What are the indications for surgery in patients with mitral regurgitation? 388
11. How is mitral regurgitation corrected? 388
12. Why is it preferable to repair rather than replace the mitral valve? 388
13. How is the mitral valve repaired? 388
14. What is the operative mortality of mitral valve repair versus mitral valve replacement? 388
15. How durable are mitral valve repairs? 388
16. What is the role of minimally invasive surgery in patients with mitral regurgitation? 388
Bonus Question 389
17. What is systolic anterior motion (SAM) of mitral valve? 389
Website 389
Bibliography 389
Chapter 77: Aortic Valvular Disease 391
1. What are the most common causes of aortic stenosis? 391
2. What is the most common anatomic anomaly in aortic stenosis? 391
3. What are the most common symptoms of aortic stenosis in adults? Infants? 391
4. What is the expected survival of patients with aortic stenosis? 391
5. What is the most feared complication of aortic stenosis? 391
6. What physical findings suggest aortic stenosis? 391
7. What are the typical findings of aortic stenosis on chest radiographs and electrocardiogram (ECG)? 391
8. How is the diagnosis of aortic stenosis confirmed? 391
9. When is cardiac catheterization indicated in patients with aortic stenosis confirmed by echocardiography? 392
10. When is an operation indicated for aortic stenosis? 392
11. What should be given to all patients with AS undergoing elective clean-contaminated or contaminated surgery?... 392
12. What hematologic disorder occurs in patient with severe aortic stenosis? 392
13. Can aortic valvotomy be used to treat aortic stenosis? 392
14. What is the Ross procedure? 392
15. What type of valvular prosthesis should be used in children requiring aortic valve replacement? 392
16. What type of valvular prosthesis should be used in adults requiring aortic valve replacement? 393
17. What are the most common causes of aortic insufficiency? 393
18. What physical findings suggest aortic insufficiency? 393
19. What is a Quincke's pulse? 393
20. How is the diagnosis of aortic insufficiency confirmed? 393
21. When is an operation indicated for aortic insufficiency? 394
22. What are the indications for aortic valve replacement in patients with infective endocarditis? 394
23. What is the operative mortality of aortic valve replacement? 394
24. What are the complications of aortic valve replacement? 394
25. What are the long-term results of aortic valve replacement? 394
26. What are the minimally invasive surgical options for AVR? 394
27. What are the potential benefits of minimally invasive AVR? 394
28. Can balloon aortic valvotomy be used for adult calcific aortic stenosis? 395
29. What are the indications for balloon valvotomy? 395
30. Is percutaneous aortic valve replacement feasible? 395
Controversies 395
31. Should the Ross procedure ever be performed? 395
32. Should a tissue valve be used in young adults between ages 15 and 30 years? 395
33. Should minimally invasive approaches to aortic valve replacement be attempted? 395
Bibliography 396
Chapter 78: Thoracic Surgery for Non-Neoplastic Disease 397
Pleural Effusion 397
1. What is a pleural effusion? 397
2. How does one determine the cause of a pleural effusion? 397
3. What is the management of a pleural effusion? 397
4. What does an air-fluid level on an initial chest radiograph indicate? 397
Empyema 398
5. What is an empyema, and what causes it? 398
6. What are the three stages of empyema development? 398
7. How is an empyema diagnosed? 398
8. How should an empyema be treated? 398
9. What is a decortication? 398
10. What are the complications of an empyema if left untreated? 398
Infections and Tuberculosis 398
11. What is a lung abscess, and how is it treated? 398
12. What are the clinical manifestations of pulmonary tuberculosis? 399
13. How is the diagnosis of pulmonary tuberculosis made? 399
14. What is the current medical treatment for active tuberculosis? 399
15. What are the indications for surgery in patients with tuberculosis? 399
16. What is MOTT, and what is the role of surgery with this disease? 399
Website 400
Bibliography 400
Chapter 79: Lung Cancer 401
1. How common is lung cancer? 401
2. What risk factors are thought to be important in the development of lung cancer? 401
3. Do genes and heredity play a role in lung cancer? 401
4. What are the major histologic types of lung cancer? 402
5. Is lung cancer screening effective? 402
6. How do patients with lung cancer present? 403
7. What is a paraneoplastic syndrome? 403
8. Does the staging system for lung cancer have prognostic and therapeutic importance? 403
9. Describe the work-up of a patient with a mass on chest radiograph. 403
10. How are patients with lung cancer treated? 404
11. Do chemotherapy radiation therapy have a place in the therapy of lung cancer? 404
12. What is the survival rate of patients treated for non-small cell lung cancer at 5 years? 405
13. What is mediastinoscopy? 405
14. What are the indications for mediastinoscopy? 405
15. Is malignant pleural effusion or recurrent nerve involvement with tumor an absolute contraindication to surgical resection for lung cancer? 405
Website 405
Bibliography 405
Chapter 80: Solitary Pulmonary Nodule 407
1. What is a solitary pulmonary nodule? 407
2. What causes a solitary pulmonary nodule? 407
3. How does a solitary pulmonary nodule present? 407
4. How frequently does a solitary pulmonary nodule represent metastatic disease? 407
5. Can a tissue sample be obtained by fluoroscopic or CT-guided needle biopsy? 407
6. Are radiographic findings important? 407
7. What social or clinical findings suggest that a nodule is malignant rather than benign? 408
8. What is the most valuable bit of historic data? 408
9. If a patient presents with a treated prior malignancy and a new solitarypulmonary nodule, is it safe to assume that the new nodule representsmetastatic disease? 408
10. How should a solitary pulmonary nodule be evaluated? 408
11. If the lesion proves to be cancer, what is the appropriate surgical therapy? 409
Website 409
Bibliography 409
Chapter 81: Dissecting Aortic Aneurysm 410
1. Why is the term dissecting aortic aneurysm really incorrect? 410
2. When should the diagnosis be entertained? 410
3. After the diagnosis is entertained, how should the patient be managed? 410
4. What is the most significant diagnostic clue on physical examination? 410
5. Which chest radiograph findings are helpful in diagnosis? 410
6. How is the diagnosis confirmed? What are the best diagnostic studies? 410
7. What are the types of dissection? 411
8. Who cares whether a dissection involves the ascending (type A) or descending (type B) aorta? 411
9. What is the key to medical management? 411
10. What are the principles and advantages of surgical management of acute aortic dissection? 411
11. What are the operative complications? 412
12. What are the long-term results? 412
Controversies 412
13. Which is preferred: surgical or medical management of descending dissections? 412
14. What is the preferred management of aortic insufficiency in ascending dissections? 412
15. What is the preferred repair of descending dissections? 413
Website 413
Bibliography 413
IX. Pediatric Surgery 415
Chapter 82: Hypertrophic Pyloric Stenosis 415
1. What is hypertrophic pyloric stenosis? 415
2. Describe the typical presentation of HPS 415
3. What are the physical findings? 415
4. How is the diagnosis confirmed? 415
5. Describe the likely electrolyte abnormalities 415
6. What procedure is recommended for the correction of HPS? 415
7. What should be done if a perforation is identified? 416
8. When can postoperative feeding begin? 416
9. Describe several hypotheses about the pathogenesis of HPS 416
Bibliography 416
Chapter 83: Intestinal Obstruction of Neonates and Infants 417
1. What signs or symptoms suggest intestinal obstruction in the neonate? 417
2. What is the differential diagnosis of intestinal obstruction in neonates? 417
3. When are contrast studies of the gastrointestinal tract indicated? 417
4. Describe intestinal atresia 417
5. Distinguish duodenal atresia from other forms of intestinal atresia 417
6. Describe malrotation with midgut volvulus 418
7. Is midgut volvulus a surgical emergency? 418
8. What is meconium ileus? 418
9. What is Hirschsprung's disease? 418
10. What is intussusception? What are the therapeutic options? 419
11. What examples of neonatal obstruction can escape early detection and present later in life? 419
Bibliography 419
Chapter 84: Imperforate Anus 421
1. What is imperforate anus? 421
2. What is the VACTERL association? 421
3. How do you determine the severity of the defect in boys? 421
4. How is the lesion assessed in girls? 421
5. How are infants with anorectal malformations treated? 421
6. What is a posterior sagittal anorectoplasty (PSARP)? 421
7. What are the results after surgical reconstruction? 422
Bibliography 422
Chapter 85: Tracheoesophageal Malformations 423
1. What are tracheoesophageal fistula (TEF) and esophageal atresia (EA)? 423
2. Describe the three most common variants and the relative incidence of each type 423
3. What other anomalies occur with tracheoesophageal malformations? 423
4. Does the presence of other anomalies alter management and outcome? 423
5. Describe the clinical presentation, diagnosis, and preoperative management of patients with EA with distal TEF 423
6. Describe the clinical presentation, diagnosis, and preoperative management of isolated EA 423
7. Describe the clinical presentation, diagnosis, and preoperative management of TEF without EA 424
8. How are tracheoesophageal malformations corrected surgically? 424
9. What are the early and late complications of surgical repair? 424
Bibliography 425
Chapter 86: Congenital Diaphragmatic Hernia 426
1. What is the most common type of congenital diaphragmatic hernia? 426
2. What signs and symptoms suggest CDH? 426
3. How is the diagnosis confirmed? 426
4. Are other anomalies associated with CDH? 426
5. What therapeutic measures should be initiated before transport or operation? 426
6. What is the \"honeymoon period\"? 426
7. Describe the operative approach 427
8. What is the most feared complication of diaphragmatic hernia? 427
9. Is PFC correctable? If so, how? 427
10. What is the survival rate for patients with CDH? 427
11. Does in utero intervention have a role in the treatment of patients with CDH? 427
Bibliography 428
Chapter 87: Abdominal Tumors 429
1. What are the most common malignant solid abdominal tumors in children? 429
2. Is it tough to differentiate Wilms' tumor from neuroblastomas clinically? 429
3. How are Wilms' tumors and neuroblastomas treated? 429
4. What are the major prognostic factors in neuroblastomas and Wilms' tumor? 429
5. What are the differences between hepatoblastomas and hepatocellular carcinomas? How are the tumors treated? 430
Controversy 430
6. Should patients with hepatoblastoma receive preoperative chemotherapy to shrink the tumors? 430
Bibliography 430
Chapter 88: Congenital Cysts and Sinuses of the Neck 431
1. What are branchial cleft anomalies? 431
2. Which anomaly is the most common? 431
3. How do patients with branchial cleft anomalies present? 431
4. What are the major operative hazards of branchial cleft remnant excision? 431
5. What is a thyroglossal duct cyst? 431
6. How do patients with thyroglossal duct cysts present? 431
7. How are thyroglossal duct cysts treated? 432
8. What is a cystic hygroma? 432
Bibliography 432
X. Transplantation 433
Chapter 91: Heart Transplantation 439
1. Who performed the first experimental heart-lung transplant? 439
2. Who performed the first experimental orthotopic heart-lung transplant? 439
3. Who developed the first surgical strategy required for human heart transplantation? 439
4. Who performed the first human heart transplant? When? 439
5. Who performed the first successful heart-lung transplant? When? 439
6. How many heart transplants are performed annually? Is the number increasing or decreasing? 439
7. What anastomoses (surgical connections) must be performed for a combined heart and lungs transplant? 439
8. What anastomoses must be performed for a heart transplant? 439
9. What is the preferred surgical technique? 439
10. Who is an acceptable cardiac donor? 440
11. Who is an acceptable cardiac recipient? 440
12. What does UNOS stand for? What is the difference between status I and status II patients? 440
13. What are the most common indications for heart transplant in adults and in children? 440
14. What percentage of potential recipients (on the transplant list) die while waiting for a heart transplant? 440
15. What are the extended donor criteria? Who would benefit most? 441
16. At what point does donor heart ischemic time influence mortality? 441
17. When is prolonged donor ischemic time appropriate? 441
18. Who pioneered hypothermic myocardial preservation? 441
19. What are the major causes of death after heart transplantation? 441
20. What is the typical infection pattern for a patient after transplant? 441
21. How is cardiac allograft rejection prevented? 441
22. Should all transplant patients receive induction therapy? 442
23. Does human leukocyte antigen mismatch influence the incidence of rejectionafter heart transplantation?... 442
24. Is ABO compatibility necessary for cardiac transplantation? 442
25. How is cardiac allograft rejection diagnosed? 442
26. What is the most serious complication of transvenous endomyocardial biopsy? 442
27. What is the incidence of cardiac allograft vasculopathy? What are the risk factors? 442
28. What is the difference between nontransplant coronary artery disease or atherosclerosis and cardiac allograft vasculopathy? 442
29. How is CAV diagnosed and treated? 442
30. Are 3-hydroxy-3-methylglutaryl coenzyme A reductase inhibitors (‘‘statin’’drugs) generally recommended for patients post cardiac transplant? 443
31. What are ventricular assist devices? 443
32. When should a heart transplant be performed after VAD implantation? 443
33. Is stem cell transplantation for heart failure a reality? What are the mechanisms of its benefit? 443
34. Is the transplanted heart denervated? 443
35. Can one heart be successfully transplanted twice? 443
36. What is \"domino heart transplant\"? 443
37. Is the heart capable of making tumor necrosis factor (TNF)? What does TNF have to do with heart transplantation? 443
38. What is the overall 30-day mortality rate after heart transplant? What is the breakdown in mortality between adult and pediatric patients? 444
39. What are the 5- and 10-year actuarial survival rates for heart transplant recipients? 444
40. What work remains to be done in heart transplantation? 444
Website 444
Bibliography 444
Chapter 92: Mechanical Circulatory Support 445
1. What are the indications for ventricular assist device (VAD)? 445
2. What are contraindications for VAD? 445
3. What workup needs to be done prior to VAD placement? 445
4. What is optimal medical management prior to VAD placement? 446
5. What predicts outcomes with VAD placement? 446
6. How long do the devices last? 446
7. How does the presence of a VAD affect transplantation? 446
8. What are the general classes of devices used today, and what are their advantages and disadvantages? 446
9. What are the perioperative issues that must be observed or addressed? 447
10. What needs to be done for anticoagulation for these devices? 447
11. What long-term management issues must be addressed? 447
12. When to transplant the bridges? 447
13. What must be evaluated before explantation? 447
Bibliography 447
Chapter 93: Lung Transplantation 448
1. Which human organ transplant was performed first, the heart or the lung? 448
2. Who performed the first human lung transplant? When? 448
3. What are the general types of lung transplants? 448
4. How many lung transplants are performed annually? Is the number increasing or decreasing? 448
5. Why is the number of combined heart-lung transplants performed annually decreasing? 448
6. Who is a candidate for a lung transplant? 448
7. What are the most common indications for single lung transplant? 448
8. What are the most common indications for a double lung transplant? 449
9. What are the most common indications for heart-lung transplant? 449
10. What is sewn to what during a single lung transplant? A double lung transplant? 449
11. Which diagnoses carry the best results for single lung transplants? 449
12. Are the survival rates different for single lung and double lung transplants? 449
13. What are the most common complications after lung transplant? 449
14. What are the major causes of death after lung transplantation? 449
15. What is primary graft dysfunction (PGD)? How is it treated? 449
16. What is the most common nonbacterial cause of pneumonia in lung transplant patients? 450
17. In addition to immune suppressive therapy, what other factors put transplanted lungs at risk for infection? 450
18. What is bronchiolitis obliterans? 450
19. How does bronchiolitis obliterans develop? 450
20. What are the risk factors for the development of bronchiolitis obliterans after lung transplant? 450
21. How is lung allograft rejection prevented? 450
22. What is the incidence of acute rejection? How is lung transplant rejection diagnosed? 450
23. What additional tests can help distinguish between acute rejection and infection? 450
24. Describe the phenomenon of chimerism in transplantation 451
25. Does chimerism develop in the heart and the lungs? 451
26. Why is chimerism exciting? 451
27. What are the major types of preservation solutions for heart and lung grafts? 451
28. What are the main differences in composition between Euro-Collins and University of Wisconsin solutions and Perfadex and Celsior? 451
29. What percentage of pulmonary blood flow goes to the transplanted lung after single lung transplant? 451
30. Is cardiopulmonary bypass required for lung transplantation? 452
31. Is living-related lung transplant possible? 452
32. How can stem cells improve pulmonary function before and after lung transplantation? 452
33. What is lung volume reduction surgery? How may it be important to patients on the lung transplant waiting list? 452
34. Who is the best candidate for lung volume reduction surgery? 452
35. What are the contraindications to lung reduction surgery? 452
36. What are the 1-year, 3-year, and 5-year actuarial survival rates for single lung retransplants? 452
37. Is a simultaneous lung and pancreas transplant possible? 452
Website 453
Bibliography 453
XI. Urology 455
Chapter 94: The Surgical Approach To Infertility 455
1. How common a problem is infertility? 455
2. What are the odds that a fertile couple will become pregnant after a single episode of well-timed intercourse? 455
3. What is the best timing for intercourse if a couple is trying to conceive? 455
4. What environmental factors may play a role in male infertility? 455
5. Can a vasectomy be successfully reversed? 455
6. What is in vitro fertilization (IVF)? 455
7. What is the role of IVF in male infertility? 455
8. Can sperm obtained directly from the testicle be used to generate a pregnancy? 456
9. What is the role of sperm freezing in the treatment of infertility? 456
10. Does wearing boxer shorts versus tight underwear affect male fertility? 456
11. Because normal levels of testosterone are necessary for sperm production, is it helpful to give subfertile men additional testosterone? 456
12. What is the most common cause of male infertility? 456
13. If we can clone Dolly (a sheep derived from cloning a fully differentiated mammary cell), can we clone humans? 456
14. Is in vitro fertilization associated with an increase in genetic abnormalities? 456
15. Will giving supplemental testosterone improve male fertility? 457
16. What is cloning as it pertains to humans? 457
17. Are undescended testes associated with male infertility? 457
Website 457
Bibliography 457
Chapter 95: Urinary Calculus Disease 458
1. How common are stones of the urinary tract? 458
2. How are stones in the urinary tract diagnosed? 458
3. What are the best studies to diagnose stones? 458
4. What are the indications for admitting the patient to the hospital with stone disease? 458
5. What are the common types of urinary tract stones found in the United States? 459
6. What are the treatment options for renal stones? 459
7. What are the treatment options for ureteral stones? 459
8. What is a stent, and when are they used? 460
9. What are the potential complications of treatment for stone disease? 460
10. What is medical management of stone disease? 460
11. Can stones be dissolved? 460
12. Does a diet high in calcium increase risk of stone disease? 460
Bonus Question 460
13. Who were the \"Lithotomists\"? 460
Bibliography 461
Chapter 96: Renal Cell Carcinoma 462
1. How common is renal cell carcinoma (RCC)? 462
2. What is the etiology of RCC? 462
3. What are the signs and symptoms of RCC? 462
4. Are all solid masses in the kidney renal cell carcinoma? 462
5. What is the unique relationship between renal cell carcinoma and its vasculature? 462
6. How should suspected involvement of the vena cava be evaluated? 462
7. How is renal cell carcinoma treated? 462
8. When is nephron-sparing nephrectomy indicated in cases of renal cell carcinoma? 462
9. How is metastatic renal cell carcinoma treated? 463
Website 463
Bibliography 463
Chapter 97: Bladder Cancer 464
1. What is the incidence of transitional cell carcinoma (TCC) of the bladder? 464
2. What are the risk factors associated with bladder TCC? 464
3. What are the signs and symptoms of bladder TCC? 464
4. What is the most common histologic type of bladder cancer? 464
5. How do you evaluate a patient with hematuria and bladder mass? 464
6. How do you manage bladder TCC? 464
7. What is the recurrence rate of TCC after initial transurethral resection of bladder tumor? 464
8. How often do you expect to see a high grade muscle invasive bladder TCC? 464
9. How often should superficial lesions be followed with surveillance cystoscopy and urine cytology? 464
10. Is there a chance of concurrent urothelial cancers? 465
11. Is CIS a less aggressive type of bladder cancer? 465
12. How do you manage bladder carcinoma in situ? 465
13. What are the other indications of intravesical BCG? 465
14. What are the side effects of BCG? 465
15. When can we start the intravesical BCG treatment? 465
16. What is the most important pathological finding when choosing the treatment? 465
17. What types of urinary diversion are used with radical cystectomy? 465
18. How is metastatic bladder cancer treated? 465
19. In certain countries, TCC is not the predominant form of bladder cancer. What is the predominant histologic type? Why? 465
20. Are there any molecular markers that can be used to help predict the prognosis of bladder TCC? 465
Website 466
Bibliography 466
Chapter 98: Prostate Cancer 467
1. What is the prevalence of prostate cancer in the United States? 467
2. Do most men die with prostate cancer, rather than from it? 467
3. What are the early symptoms of prostate cancer? 467
4. What is the best screening method for prostate cancer? 467
5. How is prostate cancer diagnosed? 467
6. When is prostate biopsy indicated? 467
7. Does an elevated PSA level mean a man has prostate cancer? 467
8. What is a free PSA? 467
9. Are there any known risk factors for prostate cancer? 467
10. What is Gleason's sum? 467
11. How is clinically localized prostate cancer treated? 468
12. How is advanced metastatic prostate cancer treated? 468
13. What is the best treatment for prostate cancer? 468
Website 468
Bibliography 468
Chapter 99: Urodynamics and Voiding Dysfunction 469
1. What is urodynamics? 469
2. What is uroflowmetry? 469
3. What is benign prostatic hyperplasia (BPH)? 469
4. What is an American Urological Association Symptom Score? 469
5. What are the main functions of the LUT? 469
6. What are the control mechanisms for LUT function? 469
7. What is the role of the autonomic nervous system in the function of the LUT? 469
8. Is there a better way to memorize this function? 470
9. What is the role of the somatic nervous system in the function of the LUT? 470
10. What is the bulbocavernosal reflex? 470
11. What is the most common cause of urinary incontinence in the geriatric population? 470
12. What is spinal shock? What type of urinary dysfunction does it cause? 470
13. What is autonomic dysreflexia? How is it treated? 470
14. What type of bladder dysfunction is frequently seen in patients with diabetes? 470
15. What type of bladder dysfunction is frequently seen in patients with multiple sclerosis (MS)? 471
16. Which sacral roots control the micturition physiology? 471
17. What are the causes of urinary retention after abdominal or pelvic surgery? 471
18. What is Ogilvie's syndrome? 471
Website 471
Bibliography 471
Chapter 100: Pediatric Urology 472
1. A healthy 3-year-old girl develops a febrile urinary tract infection. How should she be evaluated? 472
2. What is vesicoureteral reflux disease? 472
3. Is VUR damaging to the kidney? 472
4. What are the indications for surgical correction of VUR? 472
5. What is the most common cause of antenatal hydronephrosis? 472
6. What is the most common cause of UPJ obstruction? 472
7. Can UPJ obstruction resolve spontaneously? What are the indications for pyeloplasty? 472
8. What is the Meyer-Weigert law? 473
9. What is a ureterocele? 473
10. What is an ectopic ureter? 473
11. What is the most common presenting symptom in a girl with an ectopic ureter? 473
12. Do boys with ectopic ureters present with incontinence? 473
13. What percentage of full-term male infants have an undescended testicle? 473
14. What is the most common location of an undescended testicle? 473
15. Why should the testicle be brought back into the scrotum? 473
16. What is the most common cause of bladder outlet obstruction in boys? In girls? 473
17. What are the urinary manifestations of posterior urethral valves? 474
18. What is a myelomeningocele? What are its urologic consequences? 474
19. What is the most common cause of ambiguous genitalia in the newborn? 474
20. What diagnostic evaluation should be performed in any male infant presenting with hypospadias and cryptorchidism? 474
21. What is the most common solid renal mass in infancy? In childhood? 474
Bibliography 474
XII. Health Care 475
Chapter 101: Can Health Care Be Reformed? 475
1. Is health care reform an oxymoron? 475
2. What is fee for service? 475
3. What is discounted fee for service? 475
4. Is there a difference between hospital costs and hospital charges? 475
5. What are fixed costs? 475
6. What are actual costs? 475
7. Is hospital accounting a precisely scientific and objective analysis of financial data? 475
8. What is health insurance? 476
9. What are health maintenance organizations? 476
10. Initially, a lot of physicians did not like HMOs. Why? 476
11. Why are physicians fiercely independent? 476
12. Is that good? 476
13. Do HMO administrators really dictate how physicians manage their patients? 476
14. Do physicians follow these clinical pathways? 476
15. What do HMO managers do? 476
16. Do physicians welcome this kind of scrutiny? 476
17. What is a preferred provider organization (PPO)? 476
18. Is health care expensive? 477
19. So what is the problem? 477
20. Does big business have a solution? 477
21. What is capitation? 477
22. Why do physicians not like capitation? 477
23. Is all this change good? 477
24. Can physicians keep up with all this change? 477
25. Despite all of the medical Chicken Littles who sonorously declare that the sky is falling, is medicine (and even more clearly, surgery)... 477
Bibliography 478
Chapter 102: Ethics In The Surgical Intensive Care Unit 479
1. What are the four principles of medical ethics? 479
2. What is a do-not-resuscitate order? 479
3. What is the difference between withdrawing and withholding support? 479
4. What is an advance directive? 479
5. What is durable power of attorney? 480
6. What is a living will? 480
7. What is included in informed consent? 480
8. What are futile care and medical futility? 480
9. What are the clinical determinants of brain death? 480
10. What is a persistent vegetative state? 481
11. What is euthanasia? 481
12. Who should approach patients' families about organ donation? 481
13. What should patients' families be told when organ donation is feasible? 481
14. What is organ donation after cardiac death? 481
15. What is the role of the hospital ethics committee? 481
Bibliography 482
Chapter 103: Professionalism 483
1. What is a profession? 483
2. What are the core elements of a profession? 483
3. What is professionalism? 483
4. Why do physicians need a code of professional conduct? 483
5. What is the American College of Surgeons Code of Professional Conduct? 483
6. What are the responsibilities of professionalism described in the AmericanCollege of Surgeons Code of Professional Conduct? 483
7. Do other professional societies have a code of professional conduct? 484
8. Why do surgeons need their own code of professionalism? 484
9. What are the fundamental principles of the Code of Professional Conduct andthe codes of other professional societies? 484
10. What is the \"primacy of patient welfare\"? 484
11. What is the \"principle of patient autonomy\"? 484
12. What is the \"principle of social justice\"? 484
13. How can I apply these lofty ideas to my everyday existence on the medicalsurgicalunit? 484
Bibliography 485
Chapter 90: Kidney and Pancreas Transplantation 436
1. What are the most common indications for kidney transplantation? 436
2. Why should patients be taken off dialysis and have kidney transplants? 436
3. How long is kidney graft survival? 436
4. How long can kidneys be kept \"on ice\"? 436
5. Where is the transplanted kidney placed? 436
6. What are the indications for native nephrectomy? 436
7. Are kidney transplants from a living donor recommended? 436
8. Is donating a kidney a major operation for living donors? 437
9. What are the indications for kidney-pancreas transplantation? 437
10. Can a patient undergo pancreas transplantation before or after a kidney transplant? 437
11. How are digestive enzymes drained in a pancreas transplant? 437
12. What are some complications commonly seen with pancreas transplant? 437
Controversies 438
13. Is human leukocyte antigen matching still important? 438
14. Does pancreas transplantation halt the progression of diabetic disease? 438
15. Are islet cell transplants the answer in the future? 438
Website 438
Bibliography 438
Chapter 89: Liver Transplantation 433
1. When and where was the first liver transplant performed? 433
2. Is liver transplantation considered a safe and effective operation? 433
3. What are the most common indications for liver transplantation in the United States? 433
4. Has the most common disease requiring transplantation shifted over the years? 433
5. How is the waiting list run? 433
6. What are some of the recent advances in liver transplant surgery? 433
7. How long can a liver be kept \"on ice\"? 433
8. What are some common postoperative complications of liver transplantation? 434
9. What is the \"piggy-back\" technique? 434
10. Is living-donor liver transplantation an option? 434
11. How have transjugular intrahepatic portosystemic shunts (TIPS) improved this field of surgery? 434
Controversies 434
12. Should liver transplants be performed in individuals with alcoholic liver disease? 434
13. Should patients with hepatic malignancies have liver transplants? 435
14. Should adult-to-adult living donors be used? 435
15. Should non-heart-beating donors be used for transplant? 435
Bibliography 435
Index 487